You are on page 1of 70

JUNE 2008 BOARD EXAM QUESTIONS NP1

100 Nursing Board Exam test questions of June 2008 Nurse Licensure Examination (NLE) Nursing Practice 1 Foundation of Professional Nursing Practice Fundamentals of Nursing
Situation

Accurate computation prior to drug administration is a basic skill all nurses must

have.

1. Rudolf is diagnosed with amoebiasis and is to received Metronidazole (Flagyl) tablets 1.5 gm daily in 3 divided doses for 7 consecutive days. Which of the following is the correct dose of the drug that the client will received per oral administration? A. 1,000 mg tid B. 500 mg tid C. 1,500 mg tid D. 50 mg tid 2. Rhona, a 2 year old female was prescribed to receive 62.5 mg suspension three times a day. The available dose is 125 mg/ml. which of the following should Nurse Paolo prepare for each oral dose? A. 0.5 ml B. 1.5 ml C. 2.5 ml D. 10 ml 3. The physician ordered Potassium Chloride (KCL) in D5W 1 liter to be infused in 24 hours for Mrs. Gomez. Since Potassium Chloride is a high risk drug, Nurse Robert used an intravenous pump. Which of the following should Nurse Robert do to safely administer this drug? A. Check the pump setting every 2 hours B. Teach the client how the infusion pump operates C. Have another nurse check the infusion pump setting D. Set the alarm of the pump loud enough to be heard 4. Baby Liza, 3 months old, with a congenital heart deformity, has an order from her physician: give 3.00 cc of Lanoxin today for 1 dose only. Which of the following is the most appropriate action by the nurse? A. Clarify order with the attending physician B. Discuss the order with the pediatric heart specialist in the unit C. Administer Lanoxin intravenously as it is the usual route of administration D. Refer to the medication administration record for previous administration of Lanoxin 5. When Nurse Norma was about to administer the medications of client Lennie, the relative of Lennie told the nurse that they buy her medicines and showed the container of medications of the client. Which of the following is the most appropriate action by the nurse? A. Hold the nurse administration of the clients medication and refer to the head nurse B. Put aside the medications she prepared and instead administer the clients

medications C. Tell the client that she will inform the physician about this D. Bring the medications of the client to the nurses station and prepare accordingly

Situation You are a newly hired nurse in a tertiary hospital. You have finished your orientation program recently and you are beginning to assimilate the culture of the profession. 6. Using Benners stages of nursing expertise, you are a beginning nurse practitioner. You will rank yourself as a/an: A. competent nurse B. novice nurse C. proficient nurse D. advanced beginner 7. Benners Proficient nurse level is different from the other levels in nursing expertise in the context of having: A. the ability to organize and plan activities B. having attained an advanced level of education C. a holistic understanding and perception of the client D. intuitive and analytic ability in new situations 8. As you become socialized into the nursing culture you become a patient advocate. Advocacy is explained by the following EXCEPT: A. respecting a persons right to be autonomous B. demonstrating loyalty to the institutions rights C. shared respect, trust and collaboration in meeting health needs D. protecting and supporting another persons rights 9. Modern day nursing has led to the led development of the expanded role of the nurse as seen in the function of a: A. Clinical nurse specialist B. Critical care nurse C. community health nurse D. staff nurse 10. You join a continuing education program to help you: A. Earn credits for license renewal B. Get in touch with colleagues in nursing C. Enhance your basic knowledge D. Update your knowledge and skills related to field of interest Situation Mr. Joses chart is the permanent legal recording of all information that relates to his health care management. As such, the entries in the chart must have accurate data. 11. Mr. Joses chart contains all information about his health care. The functions of records include all except: A. means of communication that health team members use to communicate their contributions to the clients health care

B. the clients record also shows a document of how much health care agencies will be reimbursed for their services C. educational resource for student of nursing and medicine D. recording of actions in advance to save time 12. An advantage of automated or computerized client care system is: A. The nursing diagnoses for a clients data can be accurately determined B. Cost of confinement will be reduced C. Information concerning the client can b easily updated D. The number of people to take care of the client will be reduced 13. Information in the patients chart is inadmissible in court as evidence when: A. The clients family refuses to have it used B. The client objects to its use C. The handwriting is not legible D. It has too many abbreviations that are unofficial 14. Nursing audit aims to: A. provide research data to hospital personnel B. study clients illness and treatment regimen closely C. compare actual nursing done to established standards D. provide information to health-care providers 15. A telephone order is given for a client in your ward. What is your most appropriate action? A. Copy the order on to the chart and sign the physicians name as close to his original signature as possible B. Repeat the order back to the physician, copy onto the order sheet and indicate that it is a telephone order C. Write the order in the clients chart and have the head nurse co-sign it D. Tell the physician that you can not take the order but you will call the nurse supervisor Situation Errors while providing nursing care to patients must be avoided and minimized at all time. Effective management of available resources enables the nurse to provide safe, quality patient care. 16. In the hospital where you work, increased incidence of medication error was identified as the number one problem in the unit. During the brainstorming session of the nursing service department, probable causes were identified. Which of the following is process related? A. interruptions B. use of unofficial abbreviations C. lack of knowledge D. failure to identify client 17. Miscommunication of drug orders was identified as a probable cause of medication errors. Which of the following is safe medication practice related to this? A. Maintain medication in its unit dose package until point of actual administration B. Note both generic and brand name of the medication in the Medication

Administration Method C. Only officially approved abbreviations maybe used in prescription orders D. Encourage clients to ask question about their medications. 18. The hospital has an ongoing quality assurance program. Which of the following indicates implementation of process standards? A. The nurses check clients identification band before giving medications B. The nurse reports adverse reaction to drugs C. Average waiting time for medication administration is measured D. The unit has well ventilated medication room 19. Which of the following actions indicate that Nurse Jerome is performing outcome evaluation of quality care? A. Interviews nurses for comments regarding staffing B. Measures waiting time for clients per nurses call C. Checks equipment for its calibration schedule D. Determines whether nurses perform skin assessment every shift 20. An order for a client was given and the nurse in charge of the client reports that she has no experience of doing the procedure before. Which of the following is the most appropriate action of the nurse supervisor? A. Assign another nurse to perform the procedure B. Ask the nurse to find way to learn the procedure C. Tell the nurse to read the procedure manual D. Do the procedure with the nurse Situation Oral care is an important part of hygienic practices and promoting client comfort. 21. An elderly client, 84 years old, is unconscious. Assessment of the mouth reveals excessive dryness and presence of sores. Which of the following is BEST to use for oral care? A. lemon glycerine B. hydrogen peroxide C. Mineral oil D. Normal saline solution 22. When performing oral care to an unconscious client, which of the following is a special consideration to prevent aspiration of fluids into the lungs? A. Put the client on a sidelying position with head of bed lowered B. Keep the client dry by placing towel under the chin C. Wash hands and observe appropriate infection control D. Clean mouth with oral swabs in a careful and an orderly progression 23. The advantages of oral care for a client include all of the following, EXCEPT: A. decreases bacteria in the mouth and teeth B. reduces need to use commercial mouthwash which irritate the buccal mucosa C. improves clients appearance and self-confidence D. improves appetite and taste of food

24. A possible problem while providing oral care to unconscious clients is the risk of fluid aspiration to lungs. This can be avoided by: A. Cleaning teeth and mouth with cotton swabs soaked with mouthwash to avoid rinsing the buccal cavity B. swabbing the inside of the cheeks and lips, tongue and gums with dry cotton swabs C. use fingers wrapped with wet cotton washcloth to rub inside the cheeks, tongue, lips and ums D. suctioning as needed while cleaning the buccal cavity 25. Your client has difficulty of breathing and is mouth breathing most of the time. This causes dryness of the mouth with unpleasant odor. Oral hygiene is recommended for the client and in addition, you will keep the mouth moistened by using: A. salt solution B. water C. petroleum jelly D. mentholated ointment Situation - Mr. Martin, 71 years old was suddenly rushed to the hospital because of severe chest pain. On admission, he was diagnosed to have acute myocardial infarction and was placed in the ICU. 26. While in the ICU, he executes the document tat list the medical treatment he chooses to refuse in case his condition becomes severe to a point that he will be unable to make decisions for himself. This document is: A. living will B. informed consent C. last will and testament D. power of attorney 27. After one day, the patients condition worsened and feeling hopeless. He requested the nurse to remove the oxygen. The nurse should: A. follow the patient because it is his right to die gracefully B. follow the patient as it is his right to determine the medical regimen he needs C. refuse the patient and encourage him to verbalize hid feelings D. refuse the patient since euthanasia is not accepted in the Philippines 28. Euthanasia is an ethical dilemma which confronts nurses in the ICU because: A. the choices involved do not appear to be clearly right or wrong B. a clients legal right co-exist with the nurses professional obligation C. decisions has to be made based on societal norms. D. decisions has to be mad quickly, often under stressful conditions 29. A nurse who supports a patient and familys need to make decisions that is right for them is practicing which of the following ethical principles? A. Autonomy B. confidentiality C. privacy D. truthfulness

30. Mr. Martin felt better after 5 days but recognizing the severity of his illness, he executes a document authorizing the wife to transact any form of business in his behalf in addition to all decisions relative to his confinement his document is referred to as: A. power f attorney B. living will C. informed consent D. medical records Situation - Miss Castro was recently appointed chief nurse of a 50-bed government hospital in Valenzuela. On her first day of duty, she tried to remember the elements of administration she learned from her basic nursing education. 31. One of the first things Ms. Castro did was to engage her until in objective writing, formulating goals and philosophy of nursing service. Which activities are MOST appropriately described to which elements of administration? A. planning B. controlling C. directing d. organizing 32. In recognizing the Department of Nursing, she comes up with the organizational structure defining the role and function of the different nursing positions and line-up the position with qualified people. This is included in which element of administration: A. monitoring B. evaluation C. organizing D. planning 33. After one month, she and her management committee assess the regulatory measures taken and correct whatever discrepancies are found. This is part of which element of administration: A. monitoring B. organizing C. evaluation D. planning 34. Revaluation and administrative process is BEST described as: A. a continuing process of seeing that performance meets goals and targets B. obtaining commitment of members to do better C. informing personnel how well and how much improvement has been made D. follow-up of activities that have been studied 35. In all of the various administrative functions, which of the following management skill is demanded efficiently and effectively of Ms. Castro? 1. Decision making skills 3. Auditing skills 2. Forecasting skills 4. Communications skills

A. 2 & 3 B. 1 & 4 C. 1 & 2 D. 2 & 4 Situation - Health is wealth specifically in this time of the century. The nurse is trained to promote well being of the people. 36. How does a nurse promote ones well being? A. periodic travels for rest and recreation B. faithful and observance of healthy simple lifestyle C. run away from polluted, stressful areas D. avoid sleepless, over fatigue nights 37. The nurse can be involved with health promotion as a significant person in helping the family:

A. become a better family B. prevent disease C. control their symptoms D. modify health promotive behaviors 38. The nurse should NOT leave medication at the bedside because: a. the bedside table is not sterile b. it is convenient for the nurse c. the nurse will not be able to accurately document that the patient actually took the medication d. the patient may forget to take it. 39. Non-pharmacologic pain management includes all the following EXCEPT: a. relaxation techniques b. massage c. use of herbal medicines d. body movement 40. When assessing a clients blood pressure, the nurse finds it necessary to recheck the reading. How many seconds after deflating the cuff should the nurse wait before rechecking the pressure? a. 10 b. 30 c. 45 d. 60 Situation The use of massage and meditation to help decrease stress and pain have been strongly recommended based on documented testimonials. 41. Martha wants to do a study on this topic: Effects of massage and meditation on stress and pain. The type of research that best suits this topic is: A. Applied research B. Qualitative research

C. Basic research D. Quantitative research 42. The type of research design that does not manipulate independent variable is: A. Experimental design B. Quasi-experimental design C. Non-experimental design D. Quantitative design 43. This research topic has the potential to contribute to nursing because it seeks to A. include new modalities of care B. resolve a clinical problem C. clarify an ambiguous modality of care D. enhance client care 44. Martha does review of related literature for the purpose of A. determine statistical treatment of data research B. gathering data about what is already known or unknown about the problem C. to identify if problem can be replicated D. answering the research question 45. Clients rights should be protected when doing research using human subjects. Martha identifies these rights as follows EXCEPT: A. right of self-determination B. right to compensation C. right of privacy D. right not to be harmed Situation Richard has a nursing diagnosis of ineffective airway clearance related to excessive secretions and is at risk for infection because of retained secretions. Part of Nurse Marios nursing care plan is to loosen and remove excessive secretions in the airway. 46. Mario listens to Richards bilateral sounds and finds that congestion is in the upper lobes of the lungs. The appropriate position to drain the anterior and posterior apical segments of the lungs when Mario does percussion would be: A. Client lying on his back then flat on his abdomen on Trendelenburg position B. Client seated upright in bed or on a chair then leaning forward in sitting position then flat on his back and on his abdomen C. Client lying flat on his back and then flat on his abdomen D. Client lying on his right then left side on Trendelenburg position 47. When documenting outcome of Richards treatment Mario should include the following in his recording EXCEPT: A. Color, amount and consistency of sputum B. Character of breath sounds and respiratory rate before and after procedure C. Amount of fluid intake of client before and after the procedure D. Significant changes in vital signs

48. When assessing Richard for chest percussion or chest vibration and postural drainage, Mario would focus on the following EXCEPT: A. Amount of food and fluid taken during the last meal before treatment B. Respiratory rate, breath sounds and location of congestion C. Teaching the clients relatives to perform the procedure D. Doctors order regarding position restrictions and clients tolerance for lying flat 49. Mario prepares Richard for postural drainage and percussion. Which of the following is a special consideration when doing the procedure? A. Respiratory rate of 16 to 20 per minute B. Client can tolerate sitting and lying positions C. Client has no signs of infection D. Time of last food and fluid intake of the client 50. The purpose of chest percussion and vibration is to loosen secretions in the lungs. The difference between the procedures is: A. Percussion uses only one hand while vibration uses both hands B. Percussion delivers cushioned blows to the chest with cupped palms while vibration gently shakes secretion loose on the exhalation cycle C. In both percussion and vibration the hands are on top of each other and hand action is in tune with clients breath rhythm D. Percussion slaps the chest to loosen secretions while vibration shakes the secretions along with the inhalation of air.

PART 2 Board Exam test questions 51 - 100


Situation A 61 year old man, Mr. Regalado, is admitted to the private ward for observation after complaints of severe chest pain. You are assigned to take care of the client. 51. When doing an initial assessment, the best way for you to identify the clients priority problem is to: A. Interview the client for chief complaints and other symptoms B. Talk to the relatives to gather data about history of illness C. Do auscultation to check for chest congestion D. Do a physical examination while asking the client relevant questions 52. Upon establishing Mr. Regalados nursing needs, the next nursing approach would be to: A. Introduce the client to the ward staff to put the client and family at ease B. Give client and relatives a brief tour of the physical set up the unit C. Take his vital signs for a baseline assessment D. Establish priority needs and implement appropriate interventions 53. Mr. Regalado says he has trouble going to sleep. In order to plan your nursing intervention you will: A. Observe his sleeping patterns in the next few days B. Ask him what he means by this statement

C. Check his physical environment to decrease noise level D. Take his blood pressure before sleeping and upon waking up 54. Mr. Regalados lower extremities are swollen and shiny. He has pitting pedal edema. When taking care of Mr. Regalado, which of the following interventions would be the most appropriate immediate nursing approach? A. Moisturize lower extremities to prevent skin irritation B. Measure fluid intake and output to decrease edema C. Elevate lower extremities for postural drainage D. Provide the client a list of food low in sodium 55. Mr. Regalado will be discharged from your unit within the hour. Nursing actions when preparing a client for discharge include all EXCEPT: A. Making a final physical assessment before client leaves the hospital B. Giving instructions about his medication regimen C. Walking the client to the hospital exit to ensure his safety D. Proper recording of pertinent data

Situation Using Maslows need theory, Airway, Breathing and Circulation are the physiological needs vital to life. The nurses knowledge and ability to identify and immediately intervene to meet these needs is important to save lives. 56. Which of these clients has a problem with the transport of oxygen from the lungs to the tissues: A. Carol with tumor in the brain B. Theresa with anemia C. Sonnyboy with a fracture in the femur D. Brigitte with diarrhea 57. You noted from the lab exams in the chart of M. Santos that he has reduced oxygen in the blood. This condition is called: A. Cyanosis B. Hypoxia C. Hypoxemia D. Anemia 58. You will do nasopharyngeal suctioning on Mr. Abad. Your guide for the length of insertion of the tubing for an adult would be: A. tip of the nose to the base of the neck B. the distance from the tip of the nose to the middle of the neck C. the distance from the tip of the nose to the tip of the ear lobe D. eight to ten inches 59. While doing nasopharyngeal suctioning on Mr. Abad, the nurse can avoid trauma to the area by: A. Apply suction for at least 20-30 seconds each time to ensure that all secretions are removed B. Using gloves to prevent introduction of pathogens to the respiratory system C. Applying no suction while inserting the catheter

D. Rotating catheter as it is inserter with gentle suction 60. Nurse Suzie is administering 12:00 PM medication in Ward 4. Two patients have to receive Lanoxin. What should Nurse Suzie do when one of the clients does NOT have a readable identification band? A. Ask the client if she is Mrs. Santos B. Ask the client his name C. Ask the room mate if the client is Mrs. Santos D. Compare the ID band with the bed tag Situation During the NUTRITION EDUCATION class discussion a 58 year old man, Mr. Bruno, shows increased interest. 61. Mr. Bruno asks what the normal allowable salt intake is. Your best response to Mr. Bruno is: A. 1 tsp of salt/day with iodine and sprinkle of MSG B. 5 gms per day or 1 tsp of table salt/day C. 1 tbsp of salt/day with some patis and toyo D. 1 tsp of salt/day but no patis and toyo 62. Your instructions to reduce or limit salt intake include all the following EXCEPT: A. eat natural food with little or no salt added B. limit use of table salt and use condiments instead C. use herbs and spices D. limit intake of preserved or processed food 63. Teaching strategies and approaches when giving nutrition education is influenced by age, sex and immediate concerns of the group. Your presentation for a group of young mothers would be best if you focus on: A. diets limited in salt and fat B. harmful effects of drugs and alcohol intake C. commercial preparation of dishes D. cooking demonstration and meal planning 64. Cancer cure is dependent on A. use of alternative methods of healing B. watching out for warning signs of cancer C. proficiency in doing breast self-examination D. early detection and prompt treatment 65. The role of the health worker in health education is to A. report incidence of non-communicable diseases to community health center B. educate as many people about warning signs of non-communicable diseases C. focus on smoking cessation projects D. monitor clients with hypertension Situation As you begin to work in the hospital where you are on probation, you are tasked to take care of a few patients. The clients have varied needs and you are expected to provide care for them.

66. An ambulatory client. Mr. Zosimo, is being prepared for bed. Which of the following nursing actions promote safety for the client? A. Turning off the lights to promote rest and sleep B. Instructing the client about the use of call system C. Raising the side rails D. Placing the bed in high position

67. Mikka, a 25 year old female client, is admitted with right lower quadrant abdominal pain. The physician diagnosed the client with acute appendicitis and an emergency appendectomy was performed. Twelve hours following surgery, the patient complained of pain. Which of the following is the most appropriate nursing diagnosis? A. Impaired mobility related to pain secondary to an abdominal incision B. Impaired movements related to pain due to surgery C. Impaired mobility related to surgery D. Severe pain related to surgery 68. You are preparing a plan of care for a client who is experiencing pain related to incisional swelling following laminectomy. Which of the following should be included in the nursing care plan? A. Encourage the client to log roll when turning B. Encourage the client to do self-care C. Instruct the client to do deep breathing exercises D. Ambulate the client in ward premises every twenty minutes 69. Mr. Lozano, 50 year old executive, is recovering from severe myocardial infarction. For the past 3 days, Mr. Lozanos hygiene and grooming needs have been met by the nursing staff. Which of the following activities should be implemented to achieve the goal of independence for Mr. Lozano? A. Involving family members in meeting clients personal needs B. Meeting his needs till he is ready to perform self-care C. Preparing a day to day activity list to be followed by client D. Involving Mr. Lozano in his care 70. Mr. Ernest Lopez is terminally ill and he choose to be at home with his family. What nursing action are best initiated to prepare the family of Mr. Lopez? A. Talk with the family members about the advantage of staying in the hospital for proper care B. Provide support to the family members by teaching ways to care for their loved one C. Convince the client to stay in the hospital for professional care D. Tell the client to be with his family Situation Myrna, a researcher, proposes a study on the relationship between health values and the health promotion activities of staff nurses in a selected college of nursing. 71. In both quantitative and qualitative research, the used of a frame of reference is required. Which of the following items serves as the purpose of a framework? A. Incorporates theories into nursings body of knowledge

B. Organizes the development of study and links the findings to nursings body of knowledge C. Provides logical structure of the research findings D. Identifies concepts and relationships between concepts 72. Myrna need to review relevant literature and studies. The following processes are undertaken in reviewing literature EXCEPT: A. locating and identifying resources B. reading and recording notes C. clarifying a research topic D. using the library 73. The primary purpose for reviewing literature is to: A. organize materials related to the problem of interest B. generate broad background and understanding of information related to the research problem of interest C. select topics related to the problem of interest D. gather current knowledge of the problem of interest 74. In formulating the research hypotheses, researcher Myrna should state the research question as: A. What is the response of the staff nurses to the health values? B. How is variable health value perceived in a population? C. Is there a significant relationship between health values and health promotion activities of the staff nurses? D. How do health values affect health promotion activities of the staff nurses? 75. The proposed study shows the relationship between the variables. Which of the following is the independent variable? A. Staff nurses in a selected college of nursing B. Health values C. Health promotion activities D. Relationship between health values and health promotion activities Situation While working in a tertiary hospital, you are assigned to the medical ward. 76. Your client, Mr. Diaz, is concerned that he can not pay his hospital bills and professional fees. You refer him to a: A. Nurse supervisor B. Social worker C. bookkeeping department D. physician 77. Mr. Magno has lung cancer and is going through chemotherapy. He is referred by the oncology nurse to a selfhelp group of clients with cancer to: A. receive emotional support B. to be a part of a research study C. provide financial assistance D. assist with chemotherapy

78. A diabetic hypertensive client, Mrs. Linao, needs a change in diet to improve her health status. She should be referred to a: A. nutritionist B. dietitian C. physician D. medical pathologist 79. When collaborating with other health team members, the best description of Nurse Ritas role is: A. encourages the clients involvement in his care B. shares and implements orders of the health team to ensure quality care C. she listens to the individual views of the team members D. helps client set goals of care and discharge 80. Nurse Rita is successful in collaborating with health team members about the care of Mr. Linao. This is because she has the following competencies: A. Communication, trust, and decision making B. Conflict management, trust, negotiation C. Negotiation, decision making D. Mutual respect, negotiation and trust Situation The practice of nursing goes with responsibilities and accountability whether you work in a hospital or in the community setting you main objective is to provide safe nursing to your clients? 81. To provide safe, quality nursing care to various clients in any setting, the most important tool of the nurse is: A. critical thinking to decide appropriate nursing actions B. understanding of various nursing diagnoses C. observation skills for data collection D. possession of in scientific knowledge about client needs 82. You ensure the appropriateness and safety of your nursing interventions while caring for various client groups by: A. creating plans of care for particular clientele B. identifying the correct nursing diagnoses for clients C. making a thorough assessment of client needs and problems D. using standards of nursing care as your criteria for evaluation 83. The effectiveness of your nursing care plan for your clients is determined by A. the number of nursing procedures performed to comfort the client B. the amount of medications administered to the client as ordered C. the number of times the client calls the nurse D. the outcome of nursing interventions based on plan of care 84. You are assigned to Mrs. Amado, age 49, who was admitted for possible surgey. She complained of recurrent pain at the right upper quadrant of the abdomen 1-2 hours after ingestion of fatty food. She also had frequent bouts of dizziness, blood pressure of 170/100, hot flashes. Which of the above symptoms would be an objective cue?

A. Blood pressure measurement of 170/100 B. Complaint of hot flashes C. Report of pain after ingestion of fatty food D. Complaint of frequent bouts of dizziness 85. While talking with Mrs. Amado, it is most important for the nurse to: A. schedule the laboratory exams ordered for her B. do an assessment of the client to determine priority needs C. tell the client that your shift ends after eight hours D. have the client sign an informed consent Situation - Eileen, 45 years old is admitted to the hospital with a diagnosis of renal calculi. She is experiencing severe flank pain, nauseated and with a temperature of 39 0C. 86. Given the above assessment data, the most immediate goal of the nurse would be which of the following?

A. Prevent urinary complication B. maintains fluid and electrolytes C. Alleviate pain D. Alleviating nausea

87. After IVP a renal stone was confirmed, a left nephrectomy was done. Her post operative order includes daily urine specimen to be sent to the laboratory . Eileen has a foley catheter attached to a urinary drainage system. How will you collect the urine specimen? A. remove urine from drainage tube with sterile needle and syringe and empty urine from the syringe into the specimen container B. empty a sample urine from the collecting bag into the specimen container C. disconnect the drainage tube from the indwelling catheter and allow urine to flow from catheter int the specimen container. D. disconnect the drainage the from the collecting bag and allow the urine to flow from the catheter into the specimen container. 88. Where would the nurse tape Eileens indwelling catheter in order to reduce urethral irritation? A. to the patients inner thigh B. to the patients lower thigh C. to the patient D. to the patient lower abdomen 89. Which of the following menu is appropriate for one with low sodium diet? A. instant noodles, fresh fruits and ice tea B. ham and cheese sandwich, fresh fruits and vegetables C. white chicken sandwich, vegetable salad and tea D. canned soup, potato salad, and diet soda 90. How will you prevent ascending infection to Eileen who has an indwelling catheter?

A. see to it that the drainage tubing touches the level of the urine B. change he catheter every eight hours C. see to it that the drainage tubing does not touch the level of the urine D. clean catheter may be used since urethral meatus is not a sterile area Situation You are assigned to take care of 10 patients during the morning shift. The endorsement includes the IV infusion and medications for these clients. 91. Mr. Felipe, 36 years old is to be given 2700 ml of D5LR to infuse for 18 hours starting at 8 am. At what rate should the IV fluid be flowing hourly? A. 100 ml per hour B. 210 ml per hour C. 150 ml per hour D. 190 ml per hour 92. Mr. Atienza is to receive 150 ml/hour of D% W IV infusion for 12 hours for a total of 1800ml. He is also losing gastric fluid which must be replaced every two hours. Between 8 am and 10 am, Mr. Atienza has lost 250 ml of gastric fluid. How much fluid should he receive at 11 am? A. 350 ml/hour B. 275 ml/hour C. 400 ml/hour D. 200 ml/hour 93. You are to apply a transdermal patch of nitoglycerin to your client. The following are important guidelines to observe EXCEPT: A. Apply to hairless clean area of the skin not subject to much wrinkling B. Patches may be applied to distal part of the extremities like forearm C. Change application and site regularly to prevent irritation of the skin D. Wear gloves to avoid any medication on your hand 94. You will be applying eye drops to Miss Romualdez. After checking all the necessary information and cleaning the affected eyelid and eyelashes, you administer the ophthalmic drops by instilling the eye drops: A. directly onto the cornea B. pressing the lacrimal duct C. into the outer third of the lower conjunctival sac D. from the inner canthus going towards the side of the eye 95. When applying eye ointment, the following guidelines apply EXCEPT: A. squeeze about 2 cm of ointment and gently close but not squeeze the eye B. apply the ointment from the inner canthus going outward of the affected eye C. discard the first bead of the eye ointment before application because the tube is likely to expel more than desired amount of ointment D. hold the tube above the conjunctival sac, do not let tip touch the conjunctiva Situation The staff nurse supervisor requests all the staff nurses to brainstorm and

learn ways to instruct diabetic clients on self-administration of insulin. She wants to ensure that there are nurses available daily to do health education classes. 96. The plan of the nurse supervisor is an example of A. in service education process B. efficient management of human resources C. increasing human resources D. primary prevention 97. When Mrs. Guevarra, a nurse, delegates aspects of the clients care to the nurse-aide who is an unlicensed staff, Mrs. Guevarra A. makes the assignment to teach the staff member B. is assigning the responsibility to the aide but not the accountability for those tasks C. does not have to supervise or evaluate the aide D. most know how to perform task delegated 98. Connie, the new nurse, appears tired and sluggish and lacks the enthusiasm she had six weeks ago when she started the job. The nurse supervisor should A. empathize with the nurse and listen to her B. tell her to take the day off C. discuss how she is adjusting to her new job D. ask about her family life 99. Process of formal negotiations of working conditions between a group of registered nurses and employer is A. grievance B. arbitration C. collective bargaining D. strike 100. You are attending a certification on cardiopulmonary resuscitation (CPR) offered and required by the hospital employing you. This is A. professional course towards credits B. inservice education C. advance training D. continuing education

NP2
100 Nursing Board Exam test questions of June 2008 Nurse Licensure Examination (NLE) Nursing Practice II Community Health Nursing and Care of Mother and Child Community Health Nursing / O.B. (Obstetric) PART 1 Board Exam test questions 1 - 50
June 2008

A. B. C. D.

1. Which statement is correct regarding the use of cervical cap? It may affect the Pap Smear result It does not need to be fitted by the physician It does not require the use of spermicidal It must be removed within 24hours 2. The major components of the communication process are? A. Verbal, written, and nonverbal B. Speaker, Listener and reply C. Facial expression, tone of voice and gestures D. Message, sender, channel, Receiver and Feedback

3. The extent of burns in children are normally assessed and expressed in terms of: A. The amount of body surface that is unburned B. Percentages of total body surface area (TBSA) C. How deep the deepest burns are D. The severity of the burns on a 1 to 5 burn scale 4. The school nurse notices a child who is wearing old, dirty, poor-fitting clothes; is always hungry; has no lunch money; and is always tired. When the nurse asks the boy his tiredness, he talks of playing outside until midnight. The nurse will suspect that this child is: A. Being raised by a parent of low intelligence (IQ) B. An orphan C. A victim of child neglect D. The victim of poverty 5. Which of the following indicates the type(s) of acute renal failure? A. Four types: hemorrhagic with and without clotting, and non hemorrhagic with and without clotting B. One type: Acute C. Three types: Prerenal, intrarenal, postrenal D. Two types: Acute and subacute 6. A means of facilitating professional staff development is by building upon skills, abilities and of experience of each practitioner is called: A. The novice to expert model B. Situational leadership model C. Career enhancement D. Clinical ladder 7. Which of the following questions by the nurse would be best fit the philosophy of the nursing mutual participation model of care (NMPMC)? A. Have you brushed your childs teeth today? B. How does your child look to you today? C. Where have you been all morning? D. Do you think your childs color is worse? 8. There are numerous definitions of the word health. Which definition below is from Florence Nightingale? A. A state or a process of being and becoming an integrated and whole person B. The state of being free from illness or injury C. Being well and using every power the individual processes to the fullest extent D. A state of complete physical, social, and mental well-being and not merely the absence of diasease or infirmity. 9. Informal communication takes place when the individual talk and is best described by saying the participants: A. Are involved in a preexisting informal relationship B. Talk with slang words C. Have no particular agenda or protocol D. Are relaxed 10. Tertiary care by the home health nurse is directed toward children with: A. Problem is mobility B. Short-term needs C. Minor problems

D. Clinically apparent disease 11. The endometrium thickens during which phase of the menstrual cycle? A. Secretory phase B. Menstrual phase C. Proliferative phase D. Ischemic phase 12. A measurement tool to articulate the nursing workload for a specific patient or groups of patients over a specific period of time is called: A. Staffing pattern B. Sill mix C. Benchmaking D. Patient classification 13. The mother of a 9 month-old infant is concerned that the head circumference of her baby is greater than the chest circumference. The BEST response by the nurse is: A. These circumference normally are the same, but in some babies this just differs B. Perhaps your baby was small for gestational age or premature C. This is normal until the age of 1 year, when the chest will be greater D. Let me ask you a few questions, and perhaps we can figure out the cause of this difference 14. Which of the following approaches would work best when the nurse is communicating with an infant? A. Use an adult voice just as you would for anyone B. Communicate through the caregivers C. Allow the child time to warm up to the nurse D. Respond only after the child cries for a while 15. Evidence-based care started in medicine as a way to: A. Promote technological advances in medicine B. Incorporate collaboration within all health care disciplines C. Integrate individual experience with clinical research D. Teach medical students the art and science of medicine 16. Martina develops endometritis. What would be the best activity for her? A. lying in bed with a cold cloth on her forehead B. reading while resting in a trendelenburg position C. sitting with her feet elevated while playing cards D. walking around her room listening to music 17. Grace sustained a laceration on her leg from automobile accident. Why are lacerations of lower extremities potentially more serious among pregnant women than other? A. lacerations can provoke allergic responses due to gonadotropic hormone release B. a woman is less able to keep the laceration clean because of her fatigue C. healing is limited during pregnancy so these will not heal until after birth D. increased bleeding can occur from uterine pressure on leg veins 18. Fely has diarrhea for 2 days. She has sunken eyes, skin pinch goes back very slowly, and she is drinking poorly and irritable. She is not able to drink and there is no blood in the stool. How will you classify Felys illness? A. persistent diarrhea B. some dehydration C. severe dehydration D. no dehydration 19. The infectious agent that causes pulmonary tuberculosis is: A. mycobacterium tubercle B. Hansens bacillus C. Wuchereria bancrofti D. mycobacterium diphtheria 20. Your client, who happens to be female resident of the barangay you are covering, is an adult survivor who states: Why couldnt I make him stop the abuse? If I were stronger person, I would have been able to make him

stop. Maybe it was my fault to be abused. Based on this, which would be your most appropriate nursing diagnosis? A. social isolation B. anxiety C. chronic low self-esteem D. ineffective family coping 21. Maybelle is also scheduled to have an amniocentesis to test for fetal maturity. What instructions would you give her before this procedure? A. void immediately before the procedure to reduce your bladder size B. no more amniotic fluid forms afterward, that is why only a small amount is removed C. the intravenous fluid infused to dilate your uterus does not hurt the fetus D. the x-ray used to reveal your fetus position has no long-term effects 22. When planning teaching strategy for a pregnant woman, the nurse should do which of the following? A. give information about how the woman can manage the specific problems she identifies as relevant in her life B. omit information related to minor pains of pregnancy to prevent the woman from developing hypochondria C. provide all information to the woman in a group session with other pregnant women so she can have someone to discuss it with D. during the first prenatal visit, teach a woman the care measures necessary for health promotion throughout the pregnancy 23. Which of the following symptoms is LESS commonly noted in EARLY pregnancy? A. frequency of urination B. chills and fever C. varicosities D. Braxton Hicks Contractions 24. Bonnie, 3 months pregnant, has reported for her first prenatal visit. The nurse should instruct her to do which of the following? A. eat more dairy products and green leafy vegetables to provide an additional 300 calories each day B. increase her intake of carbohydrates-breads and sweets to prevent protein metabolism C. eat whenever she feels hungry because her body will let her know when she needs nutrients and extra calories D. limit intake of amino acids to prevent development of diabetic ketoacidosis 25. The nurse is assisting Mr. and Mrs. Cruz to prepare for childbirth in the home setting. Which of the following supplies should be readily available for the infant immediately after birth? A. vitamin K to avoid bleeding B. mild soap without perfume so as not to irritate the skin C. bulb syringe to suction fluid and mucous from the mouth D. heating lamp to avoid chilling from water evaporation 26. You were the nurse assigned to work with a child who has had whole brain radiation. You have assessed the child to be sleeping up to 20 hours a day and is having some nausea, malaise, fever and dysphasia. Based on this assessment, you are to work with the patients in which of the following areas. A. Accepting a reoccurrence of the tumor B. Dealing with the side effects of radiation therapy C. Caring for the dying child D. Accepting the imminent death of their child 27. The nurse is planning interventions for a child who has inflammatory bowel disease (IBD) with a nursing diagnosis of Nutrition: Less than body requirements. Which of the following interventions will be most helpful in resolving this nursing problem? A. Two large meals a day instead of several minerals and snacks B. Special IBD diet (diet that has been proven effective for crating IBD) C. Salt free diet, high in potassium, vitamins and minerals D. Diet as tolerated with lactose hydrolyzed milk instead of milk products and omission or highly seasoned foods and reduction of fiber. 28. Emotional intelligence consists of a number of competencies. Some of these are listed below. Which is NOT a characteristic of emotional intelligence?

A. Self-esteem B. self-awareness C. Empathy D. Self-regulation 29. Data Collection for driving and restraining forces, including costs, desirability and feasibility is a: A. People issue B. Structural issue C. Political issue D. Technology issue 30. One of four factors describing the experience of sexually abused children and the effect it has on their growth and development is stigmatization. Stigma will occur when: A. A child blames him or himself for the sexual abuse and begins to withdraw and isolate B. Newspapers and media dont keep sexual abuse private and accidentally or on purpose reveal the name of the victim. C. The child as been blamed by the abuser for his or her sexual behaviors, saying that the child asked to be touched or did not make the abuser to stop D. The childs agony is shared by other members of the family or friends when the sexual abuse becomes public knowledge

Situation Nurse Anna is a new BSN graduate and has just passed her Licensure Examination for Nurses in the Philippines. She has likewise been hired as a new Community Health Nurse in one of the Rural Health Units in their City, which of the following conditions may be acceptable TRUTHS applied to Community Health Nursing Practice. 31. Which of the following is the primary focus of community health nursing practice? A. Cure of illnesses B. Prevention of illnesses C. Rehabilitation back to health D. Promotion of health 32. In community health nursing, which of the following is our unit of service as nurses? A. The community B. The extended members of every family C. The individual members of the Barangay D. The Family 33. A very important part of the Community Health Nursing Assessment Process includes: A. the application of professional judgment in estimating importance of facts to family and community B. evaluation structures and qualifications of health center team C. coordination with other sectors in relation to health concerns D. carrying out nursing procedures as per plan of action 34. In community health nursing it is important to take into account the family health data coupled with an equally important need to perform ocular inspection of the area as activities which are powerful elements of: A. evaluation B. assessment C. implementation D. planning 35. The initial step in PLANNING process in order to engage in any nursing project or activities at the community level involves A. goal-setting B. monitoring C. evaluation of data D. provision of data Situation Please continue responding as a professional nurse in these other health

situations through the following questions. 36. Transmission of HIV from an infected individual to another person occurs: A. Most frequently in nurses with needlesticks B. Only if there is a large viral load in the blood C. Most commonly as a result of sexual contact D. In all infants born to women with HIV infection 37. The medical record of a client reveals a condition in which the fetus cannot pass through the maternal pelvis. The nurse interprets this as: A. Contracted pelvis B. Maternal disproportion C. Cervical insufficiency D. Fetopelvic disproportion 38. The nurse would anticipate a cesarean birth for a client who has which infection present at the onset of labor? A. Herpes-simplex virus B. Human papilloma virus C. Hepatitis D. Toxoplasmosis 39. After a vaginal examination, the nurse determines that the clients fetus is in an occiput posterior position. The nurse would anticipate that the client will have: A. A precipitous birth B. Intense back pain C. Frequent leg cramps D. Nausea and vomiting 40. The rationales for using a prostaglandin gel for a client prior to the induction of labor is to: A. Soften and efface the cervix B. Numb cervical pain receptors C. Prevent cervical lacerations D. Stimulate uterine contractions Situation Nurse Lorena is a Family Planning and Infertility Nurse Specialist and currently attends to FAMILY PLANNING CLIENTS AND INFERTILE COUPLES. The following conditions pertain to meeting the nursing needs of this particular population group. 41. Dina, 17 years old, asks you how a tubal ligation prevents pregnancy. Which would be the best answer? A. Prostaglandins released from the cut fallopian tubes can kill sperm B. Sperm can not enter the uterus because the cervical entrance is blocked. C. Sperm can no longer reach the ova, because the fallopian tubes are blocked D. The ovary no longer releases ova as there is no where for them to go. 42. The Dators are a couple undergoing testing for infertility. Infertility is said to exist when: A. a woman has no uterus B. a woman has no children C. a couple has been trying to conceive for 1 year D. a couple has wanted a child for 6 months 43. Another client named Lilia is diagnosed as having endometriosis. This condition interferes with fertility because: A. endometrial implants can block the fallopian tubes B. the uterine cervix becomes inflamed and swollen C. the ovaries stop producing adequate estrogen D. pressure on the pituitary leads to decreased FSH levels 44. Lilia is scheduled to have a hysterosalphingogram. Which of the following instructions

would you give her regarding this procedure? A. She will not be able to conceive for 3 months after the procedure B. The sonogram of the uterus will reveal any tumors present C. Many women experience mild bleeding as an after effect D. She may feel some cramping when the dye is inserted 45. Lilias cousin on the other hand, knowing nurse Lorenas specialization asks what artificial insemination by donor entails. Which would be your best answer if you were Nurse Lorena? A. Donor sperm are introduced vaginally into the uterus or cervix B. Donor sperm are injected intra-abdominally into each ovary C. Artificial sperm are injected vaginally to test tubal patency D. The husbands sperm is administered intravenously weekly 46. A child suffers a head injury in a tumbling accident in gym class. The nurses best course of action is to: A. Get the child up walking and makes sure he or she stays awake B. Leave the child and go get help C. Leave the child in the care of an older child and go get help D. Stay with the child, keep assessing and have someone call the caregivers 47. The American Academy of Pediatrics suggests that caregivers do which of the following things in regard to physical activities for preschoolers? A. Push the child to practice sports activities while they are more flexible B. Encourage a variety of physical activities in a noncompetitive environment C. Have the child engage in competitive sports to see where they excel D. Keep physical activities to a minimum until the child is in grade school. 48. Which of the following arrangements is generally considered to be the best for the parents of hospitalized infant or young child? A. Rooming in B. Separate caregiver sleeping room on the unit C. Day visits and sleeping at home D. Staying at a nearby hotel or motel 49. When one person allows the conflict to be resolved at his or her own expense, this is referred in conflict management as: A. Losing B. Winning while losing C. The win-lose approach D. The lose-win approach 50. Which of the following statements best describes acquaintance rape? A. Sexual intercourse when one person engaging in the activity is unsure about wanting to do so. B. When two people dont love each and engage in sexual activities C. When someone on a date tricks the other person into having sexual intercourse D. Sexual intercourse committed with force of the threat if force of the threat Of force without a persons contact.

100 Nursing Board Exam test questions of June 2008 Nurse Licensure Examination (NLE) Nursing Practice II Community Health Nursing and Care of Mother and Child Community Health Nursing / O.B. (Obstetric) PART 1 Board Exam test questions 51 - 100
51. A public health nurse would instruct a pregnant woman to notify the physician immediately if which of the following symptoms occur during pregnancy? A. Presence of dark color in the neck B. Increased vaginal discharge C. Swelling of the face D. Breast tenderness

52. A woman who is week 9 pregnant comes to the Health center with moderate bright red vaginal bleeding. On physical examination, the physician finds the clients cervix 2 cm dilated. Which term best describes the clients condition? A. missed abortion B. Incomplete abortions C. Inevitable abortion D. Threatened abortion 53. In a big government hospital, Nurse Pura is taking care of a woman with diagnosis of abruption placenta. What complication of this condition is most concerned to Nurse Pura? A. Urinary tract infection B. Pulmonary embolism C. Hypocalcemia D. Disseminated intravascular coagulation 54. Which of the following findings on a newly delivered womans chart would indicate she is at risk for developing postpartum hemorrhage? A. Post-term delivery B. Epidural anesthesia C. Grand multiparity D. Premature rupture of membrane 55. Mrs. Hacienda Gracia, 35 years old postpartum client is at risk of thrombophlebitis. Which of the following nursing interventions decrease her chances of developing postpartum thrombophlebitis? A. Breastfeeding the newborn B. Early ambulation C. Administration of anticoagulant postpartum D. Immobilization and elevation of the lower extremities

Situation With increasing documented cases of CANCER the best alternative to treatment still remains to be PREVENTION. The following conditions apply. 56. Which among the following is the primary focus of prevention of cances? A. Elimination of conditions causing cancer B. Diagnosis and treatment C. Treatment at early stage D. Early detection 57. In the prevention and control of cancer, which of the following activities is the most important function of the community health nurse? A. Conduct community assembles B. Referral to cancer specialist those clients with symptoms of cancer C. Use the nine warning signs of cancer as parameters in our process of detection, control, and treatment modalities D. Teach woman about proper/ correct nutrition 58. Who among the following are recipients of the secondary level of care for cancer cases?

A. Those under early case detection B. Those under post case treatment C. Those under early detection D. Those scheduled for surgery 59. Who among the following are recipients of the tertiary level of care for cancer cases? A. Those under treatment B. Those under supportive care C. Those under early detection D. Those scheduled for surgery 60. In Community Health Nursing, despite the availability and the use of many equipment and devices to facilitate the job of the community health nurse, the best tool any nurse should be well be prepared to apply is a scientific approach. This approach ensures quality of care even at the community setting. This in nursing parlance is nothing less than the: A. Nursing diagnosis B. Nursing protocol C. Nursing research D. Nursing process 61. The tone and pitch of the voice, volume, inflection, speed, grunts and other vocalizations are referred to by which of the following terms? A. Paraverbal clues B. Ancillary speech C. Third element D. Enhancements 62. The plan-do-study-act cycle begins with: A. Four stages B. Five agendas C. Three questions D. Two concepts 63. During your shift, you noted one of your pregnant clients considered as waiting case manifests morning sickness and which later progressed. Which assessment finding may indicate possible developing complication? A. Maternal pulse 90 B. Trace glucose in the urine C. FHT 155 D. 1 + ketones in the urine 64. The nurse is working with a child who is going to have a bone marrow aspiration. The physician orders TAC (tetracaine, adrenaline and cocaine). Which of the following is the route of administration? A. Application to the skin, covered with a dressing prior to the procedure B. Subcutaneous C. IV using very slow drip over approximately 4 hours prior to procedure D. Nasal inhalation 65. Infant head control is judged by the:

A. Ability to hold the head without support B. Presence or absence of head lag C. Rigidity of the neck and head D. Amount of neck wrinkling 66. Which of the following could be included in the outcome criteria for a patient with a nursing diagnosis, Knowledge Deficit related to potential for altered tissue perfusion in fetus or mother related to maternal cardiovascular? A. B. C. D. Bedrest is maintained at home after the 36th wee of gestation Fetal heart rate will remain between 120 and 160 beats a minute Jugular vein distention is evident when lying at 45 degrees Maternal blood pressure maintained above the 150 systolic 67. Pregnant women should be taught to be careful to avoid accidental injury. They are prone to falls for which of the following reasons? A. additional weight from pregnancy may disturb balance when walking B. fetal activity stimulates the nerves of the legs and causes weakness C. high levels of hormones often impair judgment resulting in reckless behavior D. increased adrenalin released during pregnancy causes women to move faster than usual 68 . When a nurse uses the IMCI model, the IMCI chart uses illness classification, e.g. the pink row needs: A. no specific treatments such as antibiotics B. urgent referral C. appropriate antibiotics D. no urgent measures 69. Which of the four signs of good attachment is true in this statement? A. the chin should touch the breast while the mouth is wide open and while the lower lip is turned inward more areola is visible above than below B. the chin should touch the breast, the mouth is wide open while the lower lip turned outward and more areola visible above than below C. the chin should touch the breast while the mouth is wide open while the lower lip turned outward and more areola visible below than above D. the chin should touch the breast while the mouth is wide open and the lower lip turned inward, more areola is visible above than below 70. Which type of research inquiry investigates the issues of human complexity ( e. g. understanding the human expertise?) A. positivism B. quantitative research C. logical position D. natural inquiry 71. Carol is 15 months old and weighs 5.5 kgs and it is her initial visit. Her mother says that Carol is not eating well and unable to breastfeed, he has no vomiting, has no convulsion and not abnormally sleepy or difficult to awaken. Her temperature is 38.9 deg C. Using the integrated management of childhood illness or IMCI strategy, if you were the nurse in charge of Carol, how will you classify her illness? A. a child at a general danger sign

B. very severe febrile disease C. severe pneumonia D. severe malnutrition 72. Why are small for gestational age newborns at risk for difficulty maintaining body temperature? A. their skin is more susceptible to conduction of cold B. they are preterm so are born relatively small in size C. they do not have as many fat stored as other infants D. they are more active than usual so they throw off comes 73. Oxytocin is administered to Rita to augment labor. What are the first symptoms of water intoxication to observe for during this procedure? A. headache and vomiting B. a swollen tender tongue C. a high choking voice D. abdominal bleeding and pain 74. Which of the following treatment should NOT be considered if the child has severe dengue hemorrhagic fever? A. use plan C if there is bleeding from the nose or gums B. give ORS if there is skin Petechiae, persistent vomiting, and positive tourniquet test C. give aspirin D. prevent low blood sugar 75. In assessing the patients condition using the Integrated Management of Childhood Illness approach strategy, the first thing that a nurse should do is to: A. ask what are the childs problem B. check the patients level of consciousness C. check for the four main symptoms D. check for the general danger signs 76. A child with diarrhea is observed for the following EXCEPT: A. how long the child has diarrhea B. skin Petechiae C. presence of blood in the stool D. signs of dehydration 77. The child with no dehydration needs home treatment. Which of the following is NOT included in the care for home management at this case? A. give drugs every 4 hours B. continue feeding the child C. give the child more fluids D. inform when to return to the health center 78. Ms. Jordan, RN, believes that a patient should be treated as individual. This ethical principle that the patient referred to: A. beneficence B. nonmaleficence C. respect for person

D. autonomy 79. When patients cannot make decisions for themselves, the nurse advocate relies on the ethical principle of: A. justice and beneficence B. fidelity and nonmaleficence C. beneficence and nonmaleficence D. fidelity and justice 80. Being a community health nurse, you have the responsibility of participating in protecting the health of people. Consider this situation: Vendors selling bread with their bare hands. They receive money with these hands. You do not see them washing their hands. What should you say/do? A. Miss, may I get the bread myself because you have not washed your hands B. All of these C. Miss, it is better to use a pick up forceps/ bread tong D. Miss, your hands are dirty. Wash your hands first before getting the bread Situation 7 Nurse Joanna works as an OB-Gyne Nurse and attends to several HIGH-RISK PREGNANCIES: Particular women with preexisting or Newly Acquired illness. The following conditions apply 81. Bernadette is a 22-year old woman. Which condition would make her more prone than others to developing a Candida infection during pregnancy? A. Her husband plays golf 6 days a week B. She was over 35 when she became pregnant C. She usually drinks tomato juice for breakfast D. She has developed gestational diabetes 82. Bernadette develops a deep vein thrombosis following an auto accident and is prescribed heparin sub-Q. What should Joanna educate her about in regard to this? A. Some infants will be born with allergic symptoms to heparin B. Her infant will be born with scattered petechiae on his trunk C. Heparin can cause darkened skin in newborns D. Heparin does not cross placenta and so does not affect a fetus 83. The cousin of Bernadette with sickle-cell anemia alerted Joanna that she may need further instruction on prenatal care. What statement signifies this fact? A. Ive stopped jogging so I dont risk becoming dehydrated. B. I take an iron pill every day to help grow new red blood cells C. I am careful to drink at least eight glasses of fluid every day D. I understand why folic acid is important for red cell formation 84. Bernadette routinely takes acetylsalicylic acid (aspirin) for arthritis. Why should she limit or discontinue this toward the end of pregnancy? A. Aspirin can lead to deep vein thrombosis following birth B. Newborns develop a red rash from salicylate toxicity C. Newbors develop withdrawal headaches from salicylates D. Salicylates can lead to increased maternal bleeding at childbirth

85. Bernadette received a laceration on her leg from her automobile accident. Why are lacerations of lower extremities potentially more serious in pregnant women than others? A. Lacerations can provoke allergic responses because of gonadothropic hormone B. Increased bleeding can occur from uterine pressure on leg veins C. A woman is less able to keep the laceration clean because of her fatigue D. Healing is limited during pregnancy, so these will not heal until after birth. Situation 8 Still in your self-managed Child Health Nursing Clinic, you encounter these cases pertaining to the CARE OF CHILDREN WITH PULMONARY AFFECTIONS. 86. Josie brought her 3 months old child to your clinic because of cough and colds. Which of the following is your primary action? A. Give cotrimoxazole tablet or syrup B. Assess the patient using the chart on management of children with cough C. Refer to the doctor D. Teach the mother how to count her childs breathing

87. In responding to the care concerns of children with severe disease, referral to the hospital is of the essence especially if the child manifests which of the following? A. Wheezing B. Stop feeding well C. Fast breathing D. Difficulty to awaken 88. Which of the following is the most important responsibility of a nurse in the prevention of unnecessary deaths from pneumonia and other severe diseases? A. Giving antibiotics B. Taking of the temperature of the sick child C. Provision of Careful Assessment D. Weighing of the sick child 89. You were able to identify factors that lead to respiratory problems in the community where your health facility serve. Your primary role therefore in order to reduce morbidity due to pneumonia is to: A. Teach mothers how to recognize early signs and symptoms of pneumonia B. Make home visits to sick children C. Refer cases to hospitals D. Seek assistance and mobilize the BHWs to have a meeting with mothers 90. Which of the following is the principal focus of the CARI program of the Department of Health? A. Enhancement of health team capabilities B. Teach mothers how to detect signs and where to refer C. Mortality reduction through early detection D. Teach other community health workers how to assess patients.

Situation Among common conditions found in children especially among poor communities are ear infections/problems. The following questions apply. 91. A child with ear problem should be assessed for the following, EXCEPT: A. is there any fever? B. Ear discharge C. If discharge is present for how long? D. Ear pain 92. If the child does not have ear problem, using IMCI, what should you as the nurse do? A. Check for ear discharge B. Check for tender swellings behind the ear C. Check for ear pain D. Go to the next question, check for malnutrition 93. A ear discharge that has been present for more than 14 days can be classified as: A. mastoiditis B. chronic ear infection C. acute ear infection D. complicated ear infection 94. An ear discharge that has been present for less than 14 days can be classified as: A. chronic ear infection B. mastoiditis C. acute ear infection D. complicate ear infection 95. If the child has severe classification because of ear problem, what would be the best thing that you as the nurse can do? A. instruct mother when to return immediately B. refer urgently C. give an antibiotic for 5 days D. dry the ear by wicking Situation If the child with diarrhea registers one sign in the pink row and one in the yellow row in the IMCI Chart: 96. We can classify the patient as: A. moderate dehydration B. some dehydration C. no dehydration D. severe dehydration 97. The child with no dehydration needs home treatment. Which of the following is not included in the rules for home treatment in this case: A. continue feeding the child B. gives oresol every 4 hours C. know when to return to the health center

D. give the child extra fluids 98. A child who has had diarrhea for 14 days but has no sign of dehydration is classified as: A. severe persistent diarrhea B. dysentery C. severe dysentery D. persistent diarrhea 99. If the child has sunken eyes, drinking eagerly, thirsty and skin pinch goes back slowly, the classification would be: A. no dehydration B. moderate dehydration C. some dehydration D. severe dehydration 100. Genetic testing should be performed on a child only if: A. The parents both want it performed B. It is in the best interest of the clinic C. It is necessary for the child to survive D. No one objects

NP3
NP3 Nursing Board Exam June 2008 Answer Key 'Nursing Care of Client with Physiological and Psychosocial Alteration'
100 Nursing Board Exam test questions of June 2008 Nurse Licensure Examination (NLE) Nursing Practice III Nursing Care of Client with Physiological and Psychosocial Alteration Medical and Surgical Nursing / Psychology PART 1 Board Exam test questions 1 - 50
June 2008 NURSING PRACTICE III SET B SITUATIONAL Situation - One learns by doing especially when you practice the best methods. 1. Which action by a new nurse signifies a need for further teaching in infection control? A. The nurse places the side rails the time to an unconscious patient. B. The nurse elevates the head of the bed to check the BP C. The nurse uses her bare hands to change the dressing D. The Nurse applies oxygen catheter to the mouth. 2. You are a PM shift and about 5 patients are discharge. You noted that the orderly was looking through the items of one of the patients. Which action should you pursue?

A.Call the attention of the orderly in private. B. Ignore the situation because you are busy. C. Report this behavior to the nurse in charge. D. Monitor the situation and note whether any other items are reported missing. 3. What appropriate action should you do when you overhear the nursing attendant speaking harshly to an elderly patient? A. Try to explore the interaction with the nursing attendant concerned. B. Change the attendants assignment. C. Initiate a group discussion with all other nursing attendants. D. Discuss the matter with the patients family. 4. Disposal of Medical Records in the Government hospital needs collaboration with which of the following department: A. DOH B. MMDA C. DILG D. RMAO 5. You saw one colleague charting medication administration that she has not yet administered. After talking to her, you also report the incident to the charge nurse. The charge nurse should: A. Required the staff to submit an incident report. B. Terminate the nurse C. Charge the erring nurse with dishonesty. D. Report to the Board of Nursing.

Situation - You are assigned at the PACU 6. Which nursing diagnosis has priority among the client in the PACU? A. Acute Pain related to discomfort of wound and immobility B. Body image disturbance of wound dressing and drains. C. Ineffective airway clearance related to anesthesia. D. Knowledge deficit related to lack of information because patients are all sedated. 7. Which of the following clients at the PACU will demonstrate the effectiveness of the nurse teaching? A. The client demonstrates deep breathing, coughing, splintering and leg exercises. B. The client manifests normal temperature. C. The client sleeps well D. The client has good balance I and O. 8. Which of the following remark indicates that the clients relative understood the discharged instructions for would care? A. If the wound is painful, I will say it is normal. B. It is alright to use adhesive tape over the wound to keep it intact. C. It is ok for his pet to remain at his bedside to keep him company. D. It will report any redness or swelling of the wound. 9. You must transfer out a post-op client to her room. What would your instructions to the family include to prevent accidents? A. Report when the IV infusion is almost finished. B. Test the call system if functioning. C. Keep the room lights on for 24hrs. D. Make sure the side rails are up. 10.One of your post-op patients has a temperature of 27.9 C and was shivering. You covered him with a blanket and later took his temperature again and it is now 38.9 C. The nursing student asked you to explain the absence of shivering even if the temperature was higher. A. The patient is no longer febrile thus he is no longer chilling. B. Shivering normally disappears as temperature becomes higher. C. The body has reached its new set point thus the absence of shivering. D. The patient is feeling better.

Situation - Patients with chest tubes can be very challenging to new nurses. 11. The chest drainage of Tirso has continuous bubbling in the water seal drainage. After an hour you noticed that the bubbling stops. Which of the following condition is the possible cause of the malfunctioning sealed drainage? A. A suction being to high B. An air leak C. A tube being too small D. A tension pneumothorax 12. While you were making your endorsement, you found out the chest tube of a client was disconnected. What would be your appropriate action? A. Assist the client back to his bed and place him on the affected side. B. Cover the end of the chest tube with sterilize gauze C. Reconnect the tube to the chest tube system D. Put the end of the chest tube into a cup of sterile normal saline 13. Dr. Reyes asked you top assist him with the removal of Tirsos chest tube. You would instruct the client to: A. Continuously breathe normally during the chest tube B. Take a deep breath, exhale, and bear down C. exhale upon actual removal of the tube D. Hold breath until the chesty tube is pulled out 14. Chest tube diameter is measured or expressed in: A. French B. Gauge C. Millimeters D. Inches 15. When transporting clients with a chest tube, the system should be: A. Disconnected B. Closed C. Placed lower than the patients chest D. Placed between the legs of the client to prevent breakage Situation The preoperative nurse collaborates with the client significant others, and healthcare providers. 16. To control environment hazards in the OR, the nurse collaborates with the following departments EXCEPT: A. Biomedical division B. DOH C. Infection control committee D. DILG 17. Waste disposal poses a big problem for the hospital. Biological wastes (i.e. amputated limbs) disposal should be coordinated with the following agencies EXCEPT: A. Crematorium B. DOH C. MMDA D. DILG 18. Tess, the PACU nurse discovered that Malou, who weights 110 lbs prior to surgery, is in severe pain 8 hours after cholecystectomy. Upon checking the chart, Malou found out that she has an order of Demerol 100 mg I.M. prn for pain. Tess should verify the order with: A. Nurse supervisor B. Anesthesiologist C. Surgeon D. Intern on duty 19. Rosie, 57, who is diabetic is for debridement if incision wound. When the circulating nurse checked the present IV fluid, she found out that there is no insulin incorporated as ordered. What should the circulating nurse do? A. Double check the doctors order and call the attending MD

B. Communicate with the ward nurse to verify if insulin was incorporated or not C. Communicate with the client to verify if insulin was incorporated D. Incorporate insulin as ordered

20. The documentation of all nursing activities performed is legally and professionally vital. Which of the following should NOT be included in the patients chart? A. Presence of prosthetic devices such as dentures, artificial limbs hearing aid, etc. B. Baseline physical, emotional, and psychosocial data C. Arguments between nurses and residents regarding treatment D. Observed untoward signs and symptoms and interventions including contaminant intervening factors. Situation Technology and patients education has dramatically improved the management of the diabetic client. 21. The current insulin pumps available in the market have the following capability, EXCEPT: A. Prevent unexpected saving in blood glucose measurement B. Detect signs and symptoms of hypoglycemia and hypercalcemia C. Deliver a pre-meal bolus dose of insulin before each meal D. Deliver a continuous basal rate of insulin at 5.0 units to 2.0 units per hour 22. Discharge plan of diabetic clients include injection-site-rotation. You should emphasize that the space between sites should be: A. 6 cm B. 5 cm C. 2.5 cm D. 4 cm 23. It is critical also that a diabetic client should be educated in the possible sites if regular insulin injection. The fastest absorption rate happens at the tissue areas of: A. Gluteal area B. Deltoid area C. Anterior area D. Abdominal area 24. Self-monitoring of blood glucose (SMBG) is recommended for patients use. You will recommend this technology in the following diabetic patients EXCEPT: A. Client with proliferative retinopathy B. Unstable diabetes C. Hypoglycemia without warning D. Abdominal renal glucose threshold 25. It is necessary for a diabetic client to exercise regularly. What is the effect of regular exercise to a diabetic client? A. It burns excess glucose B. It improves insulin utilization C. It lowers glucose, improves insulin utilization, and decreases total triglyceride levels. D. It will make you fit and energized. 26. Following a fracture of the forearm or tibia, complaints of sharp, deep, unrelenting pain in the hand or foot unrelieved by analgesics or elevation of the extremity indicate which complication? a. cast syndrome b. compartment syndrome c. gangrene d. fat embolism 27. Another worthy study is the compliance to the principles of aseptic technique among the sterile OR team. Who does NOT belong to the sterile OR team? a. scrub nurse b. assistant surgeon c. x-ray technician d. surgeon

28. Circulation must be restored within 4 minutes of cardiopulmonary arrest because: a. the lungs fill with fluid b. the blood begins to coagulate c. brain cells begin to die d. irreversible kidney failure develops 29. Benny elaborated on the concept of informed consent. He determines that the nurses need more explanation if their response is one of the following: a. the consent should provide a description of alternative treatments or procedures b. the consent should offer a thorough explanation of the procedures to be done and the consequences of it. c. the consent should include the medical diagnosis and explanation of the patients condition d. the consent should describe the prognosis if the recommended care is refused. 30. Which of the assessment findings would indicate a need for possible glaucoma testing? a. intermittent loss of vision b. presence of floaters c. halos around lights d. pruritus and erythema of the conjunctiva 31. The diabetic patient asks for a snack and something to drink. The MOST appropriate choice for this client to meet nutritional needs would be which of the following: a. crackers with cheese and tea b. toast with peanut butter and cocoa c. vanilla wafers and coffee with cream d. graham crackers and warm milk 32. The patient demonstrates knowledge of the psychological response to the operation and other invasive procedure when she asks about: a. Who will be with me in the OR? b. How is the post operative pain over the site like? c. Will I be naked during the operation? d. Is it cold inside the 33. In teaching the mother the proper administration of tetracycline eye ointment, which of the following is MOST crucial? a. squirt a small amount on the inside of the infected eyes lower lid b. use clean, wet cloth to gently wipe away the pus c. wash hands before medication administration d. do not use other eye ointments or drops or put anything else in eyes. 34. The primary goal of nursing interventions after a craniotomy is: a. avoiding need for secondary surgery b. ensuring patient comfort c. preventing increased intracranial pressure d. prevention of infection 35. Mr. Salvo, a 35 year old male, has been admitted with the diagnosis of peptic ulcers. The nurse recognizes which drugs as those MOST commonly used in these patients to decrease acid secretions? a. erythromycin and flagyl b. Tagamet and zantac c. Maalox and kayexalate d. Dyazide and carafate 36. When reading the urinalysis report, the nurse recognizes this result as abnormal: a. red blood cells 15-20 b. turbid c. glucose negative d. ph 6.0 37. Mr. Hizon has had cataract surgery. Discharge teaching would include: a. wearing eye patches for the first 72 hours

b. bending at the waist acceptable if done slowly c. bending at the knees and keeping the head straight d. lifting light objects is acceptable 38. Endoscopic minimally invasive surgery has evolved from diagnostic modality to a widespread surgical technique. What department should the nurse collaborate which is unusual in conventional surgery? a. engineering department b. x-ray department c. blood bank services d. linen section 39. When the client is discharged from the hospital and is not capable of doing the needed care services, the following can assume the role, EXCEPT: a. family members b. chaplain c. significant others d. responsible caregiver 40. R.N. denotes that a nurse: A. Has satisfactory completed with the requirements to practice nursing as set by the state. B. Has a baccalaureate degree in nursing C. Is professionally ready to practice nursing D. Just a title Situation Joint Commission on Accreditation of Hospital Organization (JCAHO) patient safety goals and requirements include the care and efficient use of technology in the OR and elsewhere in the healthcare facility. 41. As the head nurse in the OR, how can you improve the effectiveness of clinical alarm systems? A. Limit suppliers to a few so that quality is maintained B. Implement a regular inventory of supplies and equipment C. Adherence to manufacturers recommendation D. Implement a regular maintenance and testing of alarm systems 42. Overdosage of medication or anesthetic can happen even with the aid of technology like infusion pumps, sphygmomanometer and similar devices/machines. As a staff, how can you improve the safety of using infusion pumps? A. Check the functionality of the pump before use B. Select your brand of infusion pump like you do with your cellphone C. Allow the technician to set the infusion pump before use D. Verify the flow rate against your computation 43. JCAHOs universal protocol for surgical and invasive procedures to prevent wrong site, wrong person, and wrong procedure/surgery includes the following, EXCEPT: A. Mark the operative site if possible B. Conduct pre-procedure verification process C. Take a video of the entire intra-operative procedure D. Conduct time out immediately before starting the procedure 44. You identified a potential risk of pre-and postoperative clients. To reduce the risk of patient harm resulting from fall, you can implement the following, EXCEPT: A. Assess potential risk of fall associated with the patients medication regimen B. Take action to address any identified risks through Incident Report (IR) C. Allow client to walk with relative to the OR D. Assess and periodically reassess individual clients risk for falling 45. As a nurse, you know you can improve on accuracy of patients identification by 2 patient identifiers, EXCEPT: A. identify the client by his/her wrist tag and verify with family members B. identify client by his/her wrist tag and call his/her by name C. call the client by his/her case and bed number

D. call the patient by his/her name and bed number Situation Team efforts is best demonstrated in the OR. 46. If you are the nurse in charge for scheduling surgical cases, what important information do you need to ask the surgeon? A. Who is your internist B. Who is your assistant and anesthesiologist, and what is your preferred time and type of surgery? C. Who are your anesthesiologist, internist, and assistant D. Who is your anesthesiologist 47. In the OR, the nursing tandem for every surgery is: A. Instrument technician and circulating nurse B. Nurse anesthetist, nurse assistant, and instrument technician C. Scrub nurse and nurse anesthetist D. Scrub and circulating nurses 48. While team effort is needed in the OR for efficient and quality patient care delivery, we should limit the number of people in the room for infection control. Who comprise this team? A. Surgeon, anesthesiologist, scrub nurse, radiologist, orderly B. Surgeon, assistants, scrub nurse, circulating nurse, anesthesiologist C. Surgeon, assistant surgeon, anesthesiologist, scrub nurse, pathologist D. Surgeon, assistant surgeon, anesthesiologist, intern, scrub nurse 49. When surgery is on-going, who coordinates the activities outside, including the family? A. Orderly/clerk B. Nurse Supervisor C. Circulating Nurse D. Anesthesiologist 50. The breakdown in teamwork is often times a failure in: A. Electricity B. Inadequate supply C. Leg work D. Communication

100 Nursing Board Exam test questions of June 2008 Nurse Licensure Examination (NLE) Nursing Practice III Nursing Care of Client with Physiological and Psychosocial Alteration Medical and Surgical Nursing / Psychology PART 2 Board Exam test questions 51 - 100

Situation Colostomy is a surgically created anus. It can be temporary or permanent, depending on the disease condition.\ 51. Skin care around the stoma is critical. Which of the following is not indicated as a skin care barriers? A. Apply liberal amount of mineral oil to the area B. Use karaya paste and rings around the stoma C. Clean the area daily with soap and water before applying bag D. Apply talcum powder twice a day 52. What health instruction will enhance regulation of a colostomy (defecation) of clients?

A. Irrigate after lunch everyday B. Eat fruits and vegetables in all three meals C. Eat balanced meals at regular intervals D. Restrict exercise to walking only 53. After ileostomy, which of the following condition is NOT expected? A. Increased weight B. Irritation of skin around the stoma C. Liquid stool D. Establishment of regular bowel movement 54. The following are appropriate nursing interventions during colostomy irrigation, EXCEPT: A. Increase the irrigating solution flow rate when abdominal cramps is felt B. Insert 2-4 inches of an adequately lubricated catheter to the stoma C. Position client in semi-Fowler D. Hang the solution 18 inches above the stoma 55. What sensation is used as a gauge so that patients with ileostomy can determine how often their pouch should be drained? A. Sensation of taste B. Sensation of pressure C. Sensation of smell D. Urge to defecate

Situation As a nurse, you should be aware and prepared of the different roles you play. 56. What role do you play when you hold all clients information entrusted to you in the strictest confidence? A. Patients advocate B. Educator C. Patients Liaison D. Patients arbiter 57. As a nurse, you can help improve the effectiveness of communication among healthcare givers by: A. Use of reminders of what to do B. Using standardized list of abbreviations, acronyms, and symbols C. One-on-one oral endorsement D. Text messaging and e-mail 58. As a nurse, your primary focus in the workplace is the clients safety. However, personal safety is also a concern. You can communicate hazards to your co-workers through the use of the following EXCEPT: A. Formal training B. Posters C. Posting IR in the bulletin board D. Use of labels and signs

59. As a nurse, what is one of the best way to reconcile medications across the continuum of care? A. Endorse on a case-to-case basis B. Communicate a complete list of the patients medication to the next provider of service C. Endorse in writing D. Endorse the routine and stat medications every shift 60. As a nurse, you protect yourself and co-workers from misinformation and misrepresentations through the following EXCEPT: A. Provide information to clients about a variety of services that can help alleviate the clients pain and other conditions B. Advising the client, by virtue of your expertise, that which can contribute to the clients well-being C. Health education among clients and significant others regarding the use of chemical disinfectant D. Endorsement thru trimedia to advertise your favourite disinfectant solution Situation You were on duty at the medical ward when Zeny came in for admission for tiredness, cold intolerance, constipation, and weight gain. Upon examination, the doctors diagnosis was hypothyroidism. 61. Your independent nursing care for hypothyroidism includes: A. administer sedative round the clock B. administer thyroid hormone replacement C. providing a cool, quiet, and comfortable environment D. encourage to drink 6-8 glasses of water 62. As the nurse, you should anticipate to administer which of the following medications to Zeny who is diagnosed to be suffering from hypothyroidism? A. Levothyroxine B. Lidocaine C. Lipitor D. Levophed 63. Your appropriate nursing diagnosis for Zeny who is suffering from hypothyroidism would probably include which of the following? A. Activity intolerance related to tiredness associated with disorder B. Risk to injury related to incomplete eyelid closure C. Imbalance nutrition to hypermetabolism D. Deficient fluid volume related to diarrhea 64. Myxedema coma is a life threatening complication of long standing and untreated hypothyroidism with one of the following characteristics. A. Hyperglycemia B. Hypothermia

C. Hyperthermia D. Hypoglycemia 65. As a nurse, you know that the most common type of goiter is related to a deficiency of: A. thyroxine B. thyrotropin C. iron D. iodine Situation Mr. Santos, 50, is to undergo cystoscopy due to multiple problems like scantly 66. You are the nurse in charge of Mr. Santos. When asked what are the organs to be examined during cystoscopy, you will enumerate as follows: A. Urethra, Kidney, Bladder B. Urethra, Bladder wall, trigone, urethral opening C. Bladder wall, uterine wall and urethral opening D. Urethral opening, urethral opening bladder. 67. You are the nurse in charge of Mr. Santos. When asked what are the organs to be examined during cystoscopy in: A. Supine B. Lithotomy C. Semi-fowler D. Trendelenburg 68. After cystoscopy, Mr. Santos asked you to explain why there is no incision of any kind. What do you yell him? A. Cystoscopy is direct visualization and examination by urologist B. Cystoscopy is done by x-ray visualization of the urinary tact C. Cystoscopy is done by using lasers on the urinary tract D. Cystoscopy is an endoscopic procedure of the unrinary tract 69. Within 24-48 hours post cystoscopy, it is normal to observe one of the following: A. Pink-tinged urine B. Distended bladder C. Signs of infection D. Prolonged hematuria 70. Leg cramps are NOT uncommon post cystoscopy. Nursing intervention includes: A. Bed rest B. Warm moist soak C. Early ambulation D. Hot sitz bath Situation A specimen is a piece of tissue of body fluid taken from the disease body organ or tissue to aid the health care team in diagnosis and effective treatment. 71. Carmen is suspected to have a left breast CA. She is scheduled in your room on 5 for frozen section. How will you prepare the specimen for laboratoty? A. Refrigerator and send it along with the days specimen

B. Send to pathology immediately without soaking solution C. Soak it in NSS D. Soak it in formalin 72. How will you label this specimen? What information was essential in the label? A. Name of the client, age, sex, hospital number B. Name of the client, age, sex C. Name, age, site, type of specimen, hospital number, doctor D. Name, doctor, type of specimen, hospital number 73. Foreign body extracted from the body like pins, needles, seeds or bullets are also considered as a specimen. You assisted in the multiple gunshot wound exploration. During the surgey send the specimen to: A. The department of pathology B. The national bureau of investigation C. The OR head nurse D. Clients family 74. A post dilation and curettage (D and C) client is for discharge. Follow-up of lab result should be part of the discharge plan. You will instruct the client to follow up result at the: A. Medical record B. Laboratory C. Doctors clinic D. Nurses station 75. You are the circulating nurse in OR 2. You have 4 thyroidectomy cases for the day. How do, you prevent switching of specimens? A. Send specimens to laboratory right away after the operation with the proper labels B. Collect all specimens and send to laboratory at the end of the day C. Label specimen at once D. Prepare 4 specimen vials first thing in the morning 76. Ms. F.X. has been admitted with right upper quadrant pain and has been placed on a low fat diet. Which of the following trays would be acceptable for her? a. liver, fried potatoes and avocado b. whole milk, rice and pastry c. ham, mashed potatoes, cream peas d. skim milk, lean fish, tapioca pudding 77. Negligence in the practice of nursing can be a ground for: a. revocation of license by the Ombudsman b. revocation of license by the BON c. revocation of license by the DOH d. revocation of license by the Nursing Department 78. One way of verifying that the right message/doctors order was communicated effectively is by: a. phrasing intelligently b. repeating the order message c. documenting

d. speaking distinctly using enough volume 79. Maria is administering a cleansing enema to a patient with fecal impaction. Before administering the enema, she should place the patient in which of the following positions? a. on the right side of the body with the head of the bed elevated 45 degrees b. left Sims position c. on the left side of the body with the head of the bed elevated 45 degree d. right Sims position 80. The functionality and integrity of instruments and medical devices used in surgical procedure, is the responsibility of the: a. surgeon b. bio-med technician c. OR nurse d. scrub nurse 81. Maria will be preparing a patient for thoracentesis. She should assist the patient to which of the following positions for the procedure? a. prone with the head turned to the side and supported by a pillow b. lying in bed on the affected side with the head of the bed elevated 45 degrees c. Sims position with the head of the bed flat d. Lying in bed on the unaffected side with the head of the bed elevated 45 degrees 82. When a patient comes to the clinic for an eye examination, the ophthalmologists administers phenylephrine 2.5% drops to: a. dilate retinal blood vessels b. anesthetize the cornea c. dilate the pupil d. removed any obstruction on the cornea 83. Which of the following statements by Ms. S.O. a chemotherapy patient with a low WBC count, a low platelet count and a hemoglobin measurement of 5.6 g would indicate the need for further teaching? a. My lips are dry an d cracking. I need some lubricant b. My husband and I have been using vaginal lubrication before my intercourse c. I check my mouth and teeth after each meal d. Ive been very constipated and need an enema 84. Patients undergoing surgery display different levels of anxiety. This is researchable. At present, it has been found out that music can decreases anxiety. What other factors can reduce anxiety that is currently done among postoperative cases? a. pre-anesthetic drugs b. pre-operative visit c. shower prior to surgery d. presence of any members of the family 85. A heavily researched topic in infection control is about the single most important procedure for preventing hospital-acquired infections. What is this procedure called? a. handwashing

b. use of scrub suite c. use of facemask d. brain washing 86. A most critical strategy in nursing communication is: a. non-verbal communication b. giving stereotyped comments c. verbal communication d. active listening 87. Mr. T.O. has undergone surgery for lyses of adhesions. He is transferred from Post Anesthesia Care Unit (PACU) to the Surgical floor, the nurse should obtain blood pressure, pulse and respiration every: a. 3 minutes b. 30 minutes c. 15 minutes d. 20 minutes 88. Mr. K, age 13, is diagnosed with chronic bronchitis. He is very dyspneic and must sit up to breath. An abnormal condition in which there is discomfort in breathing in any bed or sitting position is: a. Cheyne-stokes b. orthopnea c. eupnea d. dyspnea 89. The nurse recognizes that the MOST common causative organism in pyelonephritis is: a. E.Coli b. Klebsiella c. Candida Albicans d. Pseudomonas 90. Mr. WT is cleaning the garage and splashes a chemical to his eyes. The initial priority care following the chemical burn is to: a. irrigate with normal saline for 1 to 15 minutes b. transport to a physician immediately c. irrigate with water for 15 minutes or longer d. cover the eyes with a sterile gauze 91. Licensed nurses from foreign countries can practice nursing in the Philippines in the following condition: a. employed in state colleges and universities b. special projects with hospitals with a fixed fee c. employees by private hospitals d. medical mission whose services are free 92. A patient diagnosed with breast cancer has been offered the treatment choices of breast conservation surgery with radiation or a modified radical mastectomy. When questioned by the patient about these options, the nurse informs the patients that the lumpectomy with radiation. A. Reduces the fear and anxiety that accompany the diagnosis and treatment of cancer B. Has about the same 10 years survival rate as the modified radical mastectomy

C. Provides shorter treatment period with a fewer long term complications D. Preserves the normal appearance and sensitivity of the breast 93. The patient has a right to information regarding the operation or other invasive procedure and potential effects. This right is achieved through: a. informed consent b. preoperative visit c. charting d. doctors rounds 94. Which statement about a persons character is evident in the OR team? a. it assists in the control of feelings, thoughts and emotions in the face of difficulty b. it reflects the moral values and beliefs that are used as guides to personal behavior and actions c. it encourages the constructive use of the pleasure of the senses d. it refers to the quality of being righteous, correct, fair and impartial 95. Mrs. XP underwent D and C for dysfunctional bleeding. What is inserted vaginally to prevent postoperative bleeding? a. perineal pad b. vaginal packing c. vaginal suppository d. gelfoam Situation Mrs. Pichay is admitted to your ward. The MD ordered Prepare for thoracentesis this pm to remove excess air from the pleural cavity. 96. Which of the following nursing responsibilities is essential in Mrs. Pichay who will undergo thoracentesis? A. Support and reassure client during the procedure B. Ensure that informed consent has been signed C. Determine if client has allergic reaction to local anesthesia D. Ascertain if chest x-rays and other tests have been prescribed and completed 97. Mrs. Pichay who is for thoracentesis is assigned by the nurse to which of the following positions? A. Trendelenburg position B. Supine position C. Dorsal Recumbent position D. Orthopneic position 98. During thoracentesis, which of the following nursing intervention will be most crucial? A. Place patient in a quiet and cool room B. Maintain strict aseptic technique C. Advice patient to sit perfectly still during needle insertion until it has been withdrawn from the chest D. Apply pressure over the puncture site as soon as the needle is withdrawn 99. To prevent leakage of fluid in the thoracic cavity, how will you position the client after thoracentesis?

A. Place flat in bed B. Turn on the unaffected side C. Turn on the affected side D. On bed rest 100. Chest x-ray was ordered after thoracentesis. When your client asks what is the reason for another chest x-ray, you will explain: A. to rule out pneumothorax B. to rule out any possible perforation C. to decongest D. to rule out any foreign body

NP4
NP4 Nursing Board Exam June 2008 Answer Key 'Nursing Care of Client with Physiological and Psychosocial Alteration'
100 Nursing Board Exam test questions of June 2008 Nurse Licensure Examination (NLE) Nursing Practice IV Nursing Care of Client with Physiological and Psychosocial Alteration Medical and Surgical Nursing / Psychology PART 1 Board Exam test questions 1 - 50 June 2008
Situation Ensuring safety is one of your most important responsibilities. You will need to provide instructions and information to your clients to prevent complications. 1. Randy has chest tubes attached to a pleural drainage system. When caring for him you should: A. empty the drainage system at the end of the shift B. clamp the chest tube when suctioning C. palpate the surrounding areas for crepitus D. change the dressing daily using aseptic techniques 2. Fanny, came in from PACU after pelvic surgery. As Fannys nurse you know that the sign that would be indicative of a developing thrombophlebitis would be: A. a tender, painful area on the leg B. a pitting edema of the ankle C. a reddened area at the ankle D. pruritus on the calf and ankle 3. To prevent recurrent attacks on Terry who has acute glumerulonephritis, you should instruct her to: A. seek early treatment for respiratory infections B. take showers instead of tub bath C. continue to take the same restrictions on fluid intake D. avoid situations that involve physical activity 4. Herbert had a laryngectomy and he is now for discharge. He verbalized his concern regarding his laryngectomy tube being dislodged. What should you teach him first? A. Recognize that prompt closure of the tracheal opening may occur

B. Keep calm because there is no immediate emergency C. Reinsert another tubing immediately D. Notify the physician at once 5. When caring for Larry after an exploratory chest surgery and pneumonectomy, yourpriority would be to maintain: A. supplementary oxygen B. ventilation exchange C. chest tube drainage D. blood replacement

Situation As a nurse you need to anticipate the occurrence of complications of stroke so that life threatening situations can be prevented. 6. Wendy is admitted to the hospital with signs and symptoms of stroke. Her Glasgow Coma Scale is 6 on admission. A central venous catheter was inserted an I.V. infusion was started. As a nurse assigned to Wendy what will be your priority goal? A. Prevent skin breakdown B. Preserve muscle function C. Promote urinary elimination D. Maintain a patent airway 7. Knowing that for a comatose patient hearing is the last sense to be lost, as Judys nurse, what should you do? A. Tell her family that probably she cant hear them B. Talk loudly so that Wendy can hear you C. Tell her family who are in the room not to talk D. Speak softly then hold her hands gently 8. Which among the following interventions should you consider as the highest priority when caring for June who has hemiparesis secondary to stroke? A. Place June on an upright lateral position B. Perform range of motion exercises C. Apply antiembolic stockings D. Use hand rolls or pillows for support 9. Ivy, age 40, was admitted to the hospital with a severe headache, stiff neck and photophobia. She was diagnosed with a subarachnoid hemorrhage secondary to ruptured aneurysm. While waiting for surgery, you can provide a therapeutic environment by doing which of the following? A. honoring her request for a television B. placing her bed near the window C. dimming the light in her room D. allowing the family unrestricted visiting privileges 10. When performing a neurologic assessment on Walter, you find that his pupils are fixed and dilated. This indicated that he: A. probably has meningitis B. is going to be blind because of trauma C. is permanently paralyzed D. has received a significant brain injury Situation With the improvement in life expectancies and the emphasis in the quality of life it is important to provide quality care to our older patients. There are frequently encountered situations and issues relevant to the older patients. 11. During a meal a client with hepatitis B dislodge her IV line and bleeds on the surface of the over-the-bed table. It would be most appropriate for the nurse to instruct a housekeeper to clean the table with: A. Alcohol B. Acetone C. Ammonia D. Bleach

12. Nino is being treated with radiation therapy. What would be included in the plan of care to minimize skin damage from the radiation therapy? A. Cover the areas with thick clothing materials B. Apply a heating pad to the site C. Wash skin with water after the therapy D. Avoid applying creams and powders to the area 13. You are assigned to the following patients. Which of the following patients is most at risk for metabolic alkalosis? A. Grace, 30 years old post surgical patient who has continuous nasogastric suction B. Rachel, 55 year old who has just experienced a stroke C. Helen, 70 year old with altered level of consciousness who is unable to access water freely. D. Mary Jane a 2 year old infant receiving isotonic sodium chloride IV solution 14. Studies have shown that the highest incidence of Hodgkins disease is common among young adults. Juana, 20 years old approaches you and tells you I am worried about the mass on my neck. What should you do as a nurse? A. Tell her there is nothing to worry if it does not bother her B. Palpate Juanas neck and explain the possible cause C. Tell her Hodgkins disease is common among young adults like her D. Tell her to see a doctor 15. As a nurse, you accidentally administer 40mg of Propanolol (Inderal) to a client instead of 10mg although the client exhibits no adverse reactions to the larger dose, you should: A. Complete an incident report B. Call the hospital attorney C. Inform the clients family D. Do nothing because the clients condition is stable Situation Radiation therapy is another modality of cancer management. With emphasis on multidisciplinary management you have important responsibilities as a nurse 16. Albert is receiving external radiation therapy and he complains of fatigue and malaise. Which of the following nursing interventions would be most helpful for Albert? A. Tell him that sometimes these feelings can be psychogenic B. Refer him to the physician C. Reassures him that these feelings are normal D. Help him plan his activities 17. Immediately following the radiation teletherapy, Albert is: A. Considered radioactive fro 24hours B. Given a complete bath C. Placed on isolation for 6 hours D. Free from Radiation 18. Albert is admitted with a radiation induced thrombocytopenia. As a nurse you should observe the following symptoms: A. Petechiae, ecchymosis, epistaxis B. Weakness, easy fatigability, pallor C. Headache, dizziness, blurred vision D. Severe sore throat, bacteremia, hepatomegaly 19. What nursing diagnosis should be the highest priority? A. Knowledge deficit regarding thrombocytopenia precautions B. Activity intolerance C. Impaired tissue integrity D. Ineffective tissue perfusion, peripheral, cerebral, cardiovascular, gastrointestinal, renal 20. What intervention should you include in your care plan? A. Inspect his skin for petechiae, bruising, GI bleeding regularly B. Place Albert on strict isolation precaution C. Provide rest in between activities

D. Administer antipyretics if his temperature exceeds 38 C Situation - You are assigned to take care of four patients with different conditions. 21. KJ, who is to have a kidney transplant asks you how long will he take azathioprime (Imuran), cyclosporine and prednisone? You recognized that KJ understood the teaching when he states, I must take these medications: A. until the anastomosis heals B. during the preoperative period C. until the supply is over D. for the rest of my life 22. After the kidney transplant, you must observe KJ for signs of rejection which includes: A. fever and weight gain B. hematuria and seizure C. polyuria and jaundice D. moon face and muscle atrophy 23. FB, 28 years old with chronic renal disease plans to receive a kidney transplant. Recently, FB was told by his physician that he was a poor candidate for transplant because of his hypertension and diabetes mellitus. Now, FB tells you I want to go off dialysis, Id rather not live than to be in this treatment the rest of my life. How would you respond to him? A. leave the room and allow him to collect his thoughts B. tell FB that We all have days when we dont feel like going on C. tell FB that Treatments are only three times a week, you can live with that D. take a sit next to him and sit quietly 24. DS signed a consent form for participation in a clinical trial for implantable cardioverter defibrillators. Which statement by DS indicates the need for further teaching before true informed consent can be obtained? A. a wire from the generator will be attached to my heart B. the physician will make a small incision in my chest wall and place the generator there C. I wonder if there is another way to protect these bad rhythms D. this implanted defibrillator will protect me from those bad rhythms my heart goes into 25. KP is participating in a cardiac study in which his physician is directly involved. Which statement by KP indicates a lack of understanding about his rights as a research study participant? A. My confidentiality will not be compromised in this study B. I understand the risk associated in this study C. I can withdraw from the study anytime D. Ill have to find a new physician if I dont complete this study Situation - Foot care among patients with peripheral vascular problems is very important. 26. When teaching a client with peripheral vascular disease about foot care, you should include which instructions: A. avoid wearing canvas shoes B. avoid using a nail clipper to cut toe nails C. avoid use of cornstarch on the foot D. avoid wearing cotton socks 27. FT, who has no known history of peripheral vascular disease, comes to the emergency room complaining of sudden onset of lower leg pain. Inspection and palpation reveal absent pulses, paresthesia and a mottled, cyanotic, cold, cadaverous left calf. While the physician determines the appropriate management, you should: A. shave the affected leg in anticipation of surgery B. place a healing pad around the calf C. keep the affected leg level or slightly dependent D. elevate the affected calf as high as possible 28. Peripheral neuropathies primarily affect: A. sensory functions B. vascular functions C. optic functions D. motor functions

29. Peripheral neuropathy can BEST be controlled by: A. good glucose control B. steroid therapy C. vitamin supplement D. nothing, there is no slowing the process 30. In addition to clients with diabetes mellitus you must be aware that acute hypoglycemia can also develop in a client with: A. hypertension B. hyperthyroidism C. liver disease D. diabetes insipidus Situation - The physician has ordered 3 units of whole blood to be transfused to WQ following a repair of a dissecting aneurysm of the aorta. 31. You are preparing the first unit of whole blood for transfusion. From the time you obtain it from the blood bank, how long should you infuse it? A. 6 hours B. 1 hour C. 4 hours D. 2 hours 32. What should you do FIRST before you administer blood transfusion? A. verify client identity and blood product, serial number, blood type, cross matching results, expiration date B. verify client identity and blood product serial number, blood type, cross matching results, expiration date with another nurse C. check IV site and use appropriate BT set and needle D. verify physicians order 33. As WQs nurse, what will you do AFTER the transfusion has started? A. add the total amount of blood to be transfused to the intake and output B. discontinue the primary IV of Dextrose 5% Water C. check the vital signs every 15 minutes D. stay with WQ for 15 minutes to note for any possible BT reactions 34. WQ is undergoing blood transfusions of the first unit. The EARLIEST signs of transfusion reactions are: A. oliguria and jaundice B. urticaria and wheezing C. hypertension and flushing D. headache, chills, fever 35. In case WQ will experience an acute hemolytic reaction, what will be your PRIORITY intervention? A. immediately stop the blood transfusion, infuse Dextrose 5% in Water and call the physician B. stop the blood transfusion and monitor the patient closely C. immediately stop the BT, infuse NSS, call the physician, notify the blood bank D. immediately stop the BT, notify the blood bank and administer antihistamines Situation - The kidneys have very important excretory, metabolic, erythropoietic functions. Any disruptions in the kidneys functions can cause disease. As a nurse it is important that you understand the rationale behind the treatment regimen used. 36. PL, who is in acute renal failure, is admitted to the Nephrology Unit. The period of oliguria usually lasts for about 10 days. Which assessment parameter for kidney function will you use during the oliguric phase? A. urine output directly related to the amount of IV fluid infused B. urine output is less than 400 ml/24 hours C. urine output of 30-60 ml/hour D. no urine output, kidneys in a state of suspension 37. During the shock phase, what is the effect of the rennin-aldosterone-angiotensin system on renal function?

A. B. C. D.

increased urine output, increased absorption of sodium and water decreased urine output, decreased absorption of sodium and water increased urine output, decreased absorption of sodium and water decreased urine output, increased absorption of sodium and water

38. As you are caring for PL who has acute renal failure, one of the collaborative interventions you are expected to do is to start hypertonic glucose with insulin infusion and sodium bicarbonate to treat: A. hyperkalemia B. hypercalcemia C. hypokalemia D. hypernatremia 39. BN, 40 year old with chronic renal failure. An arteriovenous fistula was created for hemodialysis in his left arm. What diet instructions will you need to reinforce prior to his discharge? A. drink plenty of water B. restrict your salt intake C. monitor your fruit intake and eat plenty of bananas D. be sure to eat meat every meal 40. BN, is also advised not to use salt substitute in the diet because: A. salt substitute contain potassium which must be limited to prevent arrhythmias B. limiting salt substitutes in the diet prevents a buildup of waste products in the blood C. fluid retention is enhanced when salt substitutes are included in the diet D. a substance in the salt substitute interferes with fluid transfer across the capillary membrane Situation With the improvement in life expectancies and the emphasis in the quality of life it is important to provide quality care to our older patients. There are frequently encountered situations and issues relevant to the older patients. 41. Hypoxia may occur in the older patients because of which of the following physiologic changes associated with aging. A. Ineffective airway clearance B. Decreased alveolar surfaced area C. Decreased anterior-posterior chest diameter D. Hyperventilation 42. The older patient is at higher risk for incontinence because of: A. dilated urethra B. increased glomerular filtration rate C. diuretic use D. decreased bladder capacity 43. Merle, age 86, is complaining of dizziness when she stands up. This may indicate: A. dementia B. a visual problem C. functional decline D. drug toxicity 44. Cardiac ischemia in an older patient usually produces: A. ST-T wave changes B. Very high creatinine kinase level C. Chest pain radiating to the left arm D. Acute confusion 45. The most dependable sign of infection in the older patient is: A. change in mental status B. fever C. pain D. decreased breath sounds with crackles

Situation A disaster is a large-scale emergencyeven a small emergency left unmanaged may turn into a disaster. Disaster preparedness is crucial and is everybodys business. There are agencies that are in charge of ensuring prompt response. Comprehensive Emergency Management (CEM) is an integrated approach to the management of emergency programs and activities for all four emergency phases (mitigation, preparedness, response, and recovery), for all types of emergencies and disasters (natural, man-made, and attack) and for all levels of government and the private sector. 46. Which of the four phases of emergency management is defined as sustained action that reduces or eliminates long-term risk to people and property from natural hazards and their effects.? A. Recovery B. Mitigation C. Response D. Preparedness 47. You are a community health nurse collaborating with the Red Cross and working with disaster relief following a typhoon which flooded and devastated the whole province. Finding safe housing for survivors, organizing support for the family, organizing counseling debriefing sessions and securing physical care are the services you are involved with. To which type of prevention are these activities included: A. Tertiary prevention B. Primary prevention C. Aggregate care prevention D. Secondary prevention 48. During the disaster you see a victim with a green tag, you know that the person: A. has injuries that are significant and require medical care but can wait hours with threat to life or limb B. has injuries that are life threatening but survival is good with minimal intervention C. indicates injuries that are extensive and chances of survival are unlikely even with definitive care D. has injuries that are minor and treatment can be delayed from hours to Days 49. The term given to a category of triage that refers to life threatening or potentially life threatening injury or illness requiring immediate treatment: A. Immediate B. Emergent C. Non-acute D. Urgent 50. Which of the following terms refer to a process by which the individual receives education about recognition of stress reaction and management strategies for handling stress which may be instituted after a disaster? A. Clinical incident stress management B. Follow-up C. Debfriefing D. Defusion Situation As a member of the health and nursing team you have a crucial role to play in ensuring that all the members participate actively is the various tasks agreed upon. 51. While eating his meal, Matthew accidentally dislodges his IV lines and bleeds. Blood oozes on the surface of the over-bed table. It is most appropriate that you instruct the housekeeper to clean the table with: A. Acetone B. Alcohol C. Ammonia D. Bleach 52. You are a member of the infection control team of the hospital. Based on a feedback during the meeting of the committee there is an increased incidence of pseudomonas infection in the Burn Unit (3 out of 10 patients had positive blood and wound culture). What is your priority activity? A. Establish policies for surveillance and monitoring B. Do data gathering about the possible sources of infection (observation, chart review, interview).

C. Assign point persons who can implement policies. D. Meet with the nursing group working in the burn unit and discuss problem with them. 53. Part of your responsibility as a member of the diabetes core group is to getreferrals from the various wards regarding diabetic patients needing diabetes education. Prior to discharge today, 4 patients are referred to you. How would you start prioritizing your activities? A. Bring your diabetes teaching kit and start your session taking into consideration their distance from your office B. Contact the nurse in-charge and find out from her the reason for the referral C. Determine their learning needs then prioritize D. Involve the whole family in the teaching class 54. You have been designated as a member of the task force to plan activities for the Cancer Consciousness Week. Your committee has 4 months to plan and implement the plan. You are assigned to contact the various cancer support groups in your hospital. What will be your priority activity? A. Find out if there is a budget for this activity B. Clarify objectives of the activity with the task force before contacting the support groups C. Determine the VIPs and Celebrities who will be invited D. Find out how many support groups there are in the hospital and get the contact number of their president 55. You are invited to participate in the medical mission activity of your alumni association. In the planning stage everybody is expected to identify what they can do during the medical mission and what resources are needed. You thought it is also your chance to share what you can do for others. What will be your most important role where you can demonstrate the impact of nursing in health? A. Conduct health education on healthy life style B. Be a triage nurse C. Take the initial history and document findings D. Act as a coordinator Situation One of the realities that we are confronted with is our mortality. It is important for us nurses to be aware of how we view suffering, pain, illness, and even our death as well as its meaning. That way we can help our patients cope with death and dying. 56. Irma is terminally ill, she speaks to you in confidence. You now feel that Irmas family could be helpful if they knew what Irma has told to you. What should you do first? A. Tell the physician who in turn could tell the family B. Obtain Irmas permission to share the information in the family C. Tell Irma that she has to tell her family what she told you D. Make an appointment to discuss the situation with the family 57. Ruby who has been told she has terminal cancer, turns away and refuses to respond to you. You can best help her by: A. Coming back periodically and indicating your availability if she would like you to sit with her B. Insisting that Ruby should talk with you because it is not good to keep everything inside C. Leaving her alone because she is uncooperative and unpleasant to be with D. Encouraging her to be physically active as possible 58. Leo, who is terminally ill and recognizes that he is in the process of losing everything and everybody he loves, is depressed. Which of the following would best help him during his depression? A. Arrange for visitors who might cheer him B. Sit down and talk with him for a while C. Encourage him to look at the brighter side of things D. Sit silently with him 59. Which of the following statements would best indicate that Ruffy, who is dying, has accepted his impending death? A. Im ready to go.

B. I have resigned myself to dying. C. Whats the use? D. Im giving up 60. Marla, 90 years old has planned ahead for her death-philosophically, socially, financially and emotionally. This is recognized as: A. Acceptance that death is inevitable B. Avoidance of the true situation C. Denial with planning for continued life D. Awareness that death will soon occur Situation Andrea is admitted to the ER following an assault where she was hit on the face and head. She was brought to the ER by a police woman. Emergency measures were stated. 61. As Andreas nurse, what will be your priority interventions? A. Insert and intravenous catheter B. Insert an oral or nasopharyngeal airway C. Obtain arterial blood gases D. Give 100% oxygen 62. Andreas arterial blood gases respiratory acidosis. This most likely related to: A. Partially obstructed airway B. Ineffective breathing pattern C. Head injury D. Pain 63. Andrea loses consciousness. You should prepare for which of the following FIRST? A. Placement of nasogastric tube B. Placement of a second IV line C. Endotracheal intubation or surgical airway placement D. CT scan of head 64. Andreas physician gives an order of Mannitol 0.25 g/kg IV bolus for increased ICP. This is given to: A. Promote cerebral-tissue fluid movement B. Promote renal perfusion C. Correct acid-base imbalances D. Enhance renal-excretion of drugs 65. As Andreas nurse your goal is to prevent increased Intracranial Pressure (ICP). Which of the following independent nursing interventions is NOT suited for her? A. Do oropharyngeal suction every 15 minutes to prevent pulmonary aspiration? B. Keep ahead of bed 30-35 degrees elevated C. Maintain Andreas head in straight alignment and prevent hip flexion D. Prevent constipation and increases in intra-abdominal pressure 66. Andreas respiration is described as waxing and waning. You know that this rhythm of respiration is defined as: A. Biots B. Kussmauls C. Cheyne Stokes D. Eupnca 67. What do you call the triad of sign and symptoms seen in a client with increasing ICP? A. Virchows Triad B. The Chinese triad C. Cuschings Triad D. Charcots Triad 68. Which of the following is true with the Cushings Triad seen in head injuries? A. Narrowing of Pulse Pressure, Cheyne strokes respiration, Tachycardia B. Widening Pulse pressure, Irregular respiration, Bradycardia C. Hypertension, Kussmauls respiration, Tachycardia D. Hypotension, Irregular respiration, Bradycardia

69. In a client with a Cheyne stokes respiration, which of the following is the most appropriate nursing diagnosis? A. Ineffective airway clearance B. Ineffective breathing pattern C. Impaired gas exchange D. Activity Intolerance 70. You know the apnea is seen in clients with cheyne stoke respiration, APNEA is defined as: A. Inability to breath in a supine position so the patient sits up in bed to breathe. B. The patient is dead, the breathing stops C. There is an absence of breathing for a period of time usually 15 seconds or more D. A period of hypercapnea and hypoxia due to cessation of respiratory effort inspite of normal respiratory functioning Situation - Specific surgical interventions may be done when lung cancer is detected early. You have important perioperative responsibilities in caring for patients with lung cancer. 71. Horace underwent lobectomy. Which of following is the purpose of Horaces closed chest drainage post lobectomy: a. expansion of the remaining lung b. facilitation of coughing c. prevention of mediastinal shift d. promotion of wound healing 72. Which of the following observations indicates that the closed chest drainage system is functioning properly? A. absence of bubbling in the suction-control bottle B. the fluctuating movement of fluid in the long tube of the water-seal bottle during inspiration C. intermittent bubbling through the long tube of the suction control bottle. D. less than 25 ml drainage in the drainage bottle 73. Following lobectomy, you can BEST help Horace to reduce pain during the deep breathing and coughing exercises by: A. placing the patient on his/her operative side during exercises B. splinting the patients chest with both hands during the exercises C. administering the prescribed analgesic immediately prior to exercises D. providing rest for 6 hours before exercises 74. Peter underwent pneumonectomy. During the immediate post operative period, deep tracheal suction should be done with extreme caution because: A. Peter will not be able to tolerate coughing B. the tracheobronchial trees are dry C. the remaining normal lung needs minimal stimulation D. the bronchial suture line maybe traumatized 75. On which of the following positions should you place Peter who just underwent pneumonectomy? A. Prone position B. On his abdomen or on the side opposite the surgery C. On his back or on the side of surgery D. Any position is acceptable Situation Mang Felix, a 79 year old man who is brought to the Surgical Unit from PACU after a transurethral resection. You are assigned to receive him. You noted that he has a 3-way indwelling catheter for continuous fast dip bladder irrigation which is connected to a straight drainage. 76. Immediately after surgery, what would you expect his urine to be? A. Light yellow B. Amber C. Bright red D. Pinkish to red

77. The purpose of the continuous bladders irrigation is to: A. Allow continuous monitoring of the fluid output status B. Provide continuous flushing of clots and debris from the bladder C. Allow for proper exchange of electrolytes D. Ensure accurate monitoring of intake and output 78. Mang Felix informs you that he feels some discomfort on the hypogastric area and he has to void. What will be your most appropriate action? A. Remove his catheter then allow him to void his own B. Irrigate his catheter C. Tell him Go ahead and void. You have an indwelling catheter D. Assess color and rate of outflow, if there is a change refer to urologist for possible irrigation 79. You decided to check on Mang Felixs IV fluid infusion. You noted in flow rate, pallor and coldness around the insertion site. What is your assessment finding? A. Phlebitis B. Infiltration to subcutaneous tissue C. Pyrogenic reaction D. Air embolism 80. Knowing that proper documentation of assessment findings and interventions share important responsibilities of the nurse during first post operative days, which of the following is the LEAST relevant to document in the case of Mang Felix? A. Chest pain and vital signs B. Intravenous infusion rate C. Amount, color and consistency of bladder irrigation drainage D. Activities of daily living started Situation Many hospitals from bioethical review committees to ensure quality of life of patients. You are invited by the nursing service departments to participate in their bioethical review committee. You are expected to know the purpose and apply bioethical principles. 81. Which of the following is the purpose of the ethical review committee? A. Promote implementation of general standards B. Enhance health care providers liability C. Increase individuals responsibility D. Decrease Public security of health care providers action 82. Daria who is admitted to the hospital with autoimmune thrombocytopenia and a platelet count of 20,000/ml develops epistaxis and melena. Treatment with corticosteroid and immunoglobulin has not been successful. Her physician recommend splenectomy. Daria states I dont need surgery. This will go away on its own. A. Beneficence B. Justice C. Autonomy D. Advocacy 83. Zorayda is terminally ill and is experiencing sever pain. She has bone and liver metastasis. She has been on morphine for several months now. Zorayda is aware that they are having financial problems. She decides to sign DNR form,. What ethical principle did Zorayda and her family utilize as basis for their decision to sign a DNR. A. Justice B. Autonomy C. Advocacy D. Beneficence 84. Tricia, a staff nurse working in a cancer unit, is considered a role model not only by her colleagues but also by her patients. She goes out of her way to help other. She is very active in their professional organization and he practices what she teaches. What ethical principles is she practicing? A. Beneficence B. Justice C. Autonomy D. Advocacy

85. You are commuting to work riding the LRT. An older person collapsed and nobody seems to notice her. The security guard tried to make ker sit down but she remained unconscious. You saw what happened and you decided to help. With help, you brought the patient to the nearest hospital. You learned later that woman was diabetic. She was on her way To the diabetes clinic to have fasting blood sugar tested. She developed hypoglycemia. You were able to save a life. You felt good. What principle was applied? A. Advocacy B. Beneficence C. Justice D. Autonomy

Situation Ensuring safety before, during, and after a diagnostic procedure is an important responsibility of the nurse 86. To help Fernan better tolerate the bronchoscopy, you should instruct him to practice which of the following prior to the procedure? A. Clenching his fist every 2 minutes B. Breathing in and out through the nose with his mouth open C. Tensing the shoulder muscles while lying on his back D. Holding his breath periodically for 30 seconds 87. Following bronchoscopy, which of the following complains to Fernan should be noted as possible complication: A. Nausea and vomiting B. Shortness of breath and laryngeal stridor C. Blood tinged sputum and coughing D. Sore throat and hoarseness 88. Immediately after bronchoscopy, you instructed Fernan to: A. Exercise the neck muscles B. Breathe deeply C. Retrain from coughing and talking D. Clear his throat 89. Thoracentesis may be performed for cytologic study of pleural fluid. As a nurse your most important function during procedure is to: A. Keep the sterile equipment from contamination B. Assist the physician C. open and close the three-way stopcock D. Observe the patients vital signs 90. Right after thoracentesis, which of the following is most appropriate intervention? A. Instruct the patient not to cough or deep breathe for two hours B. Observe for symptoms of tightness of chest or bleeding C. Place an ice pack to the puncture site D. Remove the dressing to check for bleeding Situation Brain tumor, whether malignant or benign, has serious management implications. As a nurse, you should be able to understand the consequences of the disease and the treatment. 91. You are caring for Conrad who has a brained tumor and increased Intracranial Pressure (ICP). Which intervention should you include in your plan to reduce ICP? A. Administer bowel softener B. Position Conrad with his head turned toward the side of the tumor C. Provide sensory stimulation D. Encourage coughing and deep breathing 92. Keeping Conrads head and neck alignment results in: A. increased inthrathoracic pressure B. increased venous outflow C. decreased venous outflow

D. increased intrabdominal pressure 93. Which of the following activities may increase intracranial pressure (ICP)? A. Raising the head of the bed B. Manual hyperventilation C. Use of osmotic diuretics D. Valsalvas maneuver 94. After you asses Conrad, you suspected increased ICP. Your most appropriaterespiratory goal is to: A. maintain partial pressure of arterial O2 (PaO2) above 80 mmHg B. lower arterial pH C. prevent respiratory alkalosis D. promote CO2 elimination 95. Conrad underwent craniotomy. As his nurse, you know that drainage on a craniotomy dressing must be measured and marked. Which findings should you report immediately to the surgeon? A. Foul-smelling drainage B. Yellowish drainage C. Greenish drainage D. Bloody drainage Situation As a nurse you are expected to be competent in utilizing the nursing process in the care of your clients. 96. Getty is receiving chemotherapy for cancer. You review Gettys laboratory report and note that he has anemia. To which nursing diagnosis should you give the highest priority? A. Activity intolerance B. Impaired oral mucous membrane C. Impaired tissue perfusion, cerebral, cardiovascular, gastrointestinal D. Impaired tissue integrity 97. An immediate objective for nursing care of an overweight mildly hypertensive client with ureteral colic and hematuria is to decrease: A. Hypertension B. Pain C. Hematuria D. Weight 98. A difficult problem to deal with when caring for a patient with a partial thickness burns sustained 3 days ago is: A. Alteration in body image B. Maintenance of sterility C. Frequent dressing change D. Severe pain 99. Which outcome criterion would be appropriate for a client with a nursing diagnosis of effective airway clearance? A. Continued use of oxygen when necessary B. Breath sound clear on auscultation C. Respiratory rate of 24/min D. Presence of congestion 100. Which assessment would be most supportive of the nursing diagnosis, impaired skin integrity related to purulent drainage? A. Heart rate of 88 beats/min B. Dry and intact wound dressing C. Oral temperature D. Wound healing by first intention

NP5

NP5 Nursing Board Exam June 2008 Answer Key 'Nursing Care of Client with Physiological and Psychosocial Alteration'
100 Nursing Board Exam test questions of June 2008 Nurse Licensure Examination (NLE) Nursing Practice V Nursing Care of Client with Physiological and Psychosocial Alteration Medical and Surgical Nursing / Psychology PART 1 Board Exam test questions 1 - 50 June 2008

NURSING PRACTICE V - NURSING CARE OF CLIENT WITH PHYSIOLOGICAL AND PSYCHOSOCIAL ALTERATIONS (Part C) Situation - Felisa has a ritualistic pattern of constantly washing her hands with soap and water followed by rubbing alcohol. 1. This behavior is categorized as: A. delusional B. normal C. neurotic D. psychotic 2. A therapeutic intervention in this situation is: A. avoid limits on her behavior to release her anxiety B. call attention to her ritualistic pattern C. provide alternative behaviors to deal with increased anxiety D. ignore her behavior totally 3. The anxiety of Felisa is disabling and interferes with her job performance, interpersonal relationships and other activities of daily living. To minimize such problems, she is likely to be given: A. diazepam ( Valium ) B. haloperidol ( Haldol ) C. imipramine Hcl ( Tofranil ) D. chlorpromazine ( Thorazine ) 4. Felisa understands the effects of her medicine when expresses: A. I should watch out for signs of sore lips or sore throat B. I might have constipation C. I might feel changes in my body temperature D. I should not drive or operate machines 5. The level of anxiety that Felisa is experiencing is: A. Panic B. Severe C. Mild D. Moderate

Situation - The nurse recognizes the need to learn to cope with stress and change. She becomes interested to practice natural ways to enhance well being. 6. Lifestyle modification begins with: A. minimizing eating in fast food restaurants B. having an exercise regimen to follow regularly C. recognizing the impact of unhealthy habits

D. avoiding pollutants in the environment 7. All of these are the behavior intervention to stress management. EXCEPT: A. guided imagery B. pharmacotherapy C. meditation D. progressive muscle relaxation 8. Basic to progressive muscle relaxation is: A. focusing on an image to relax B relaxing muscles from tension C. use of industrial equipment D. stopping disturbing thoughts 9. Dietary practices are very important to the health of the Filipino family. The nurse needs to assess this lifestyle because: A. the nurse wants to change the eating patterns of the Filipino family B. the nurse knows that being overweight is a major health hazard C. the nurse wants to stop all the mainstream weight-loss diets D. the nurse has to find out what people are eating 10. A young overweight adult smokes 5-10 sticks of cigarettes/day, ambitious, looks at life as challenging and perfect and never considers change in his lifestyle, initially needs: A. commitment B. information C. skills to attempt change D. motivation Situation - Bernie and John in their late 40s have been married for 20 years and at the peak of their careers. Suddenly, Bernie discovered that her husband was falling in love with another woman. Shaken by this situation, she started to have problems sleeping and could not function well at work and at the risk of losing her job. John asked forgiveness and regret very much the hurt his wife was going through and suffered guilt feelings: 11. Bernie and John are going through a: A. situational crisis B. developmental crisis C. anticipated crisis D. both developmental and situational crisis 12. All of these are characteristics of crisis EXCEPT: A. a hazardous or threatening event occurs B. it has a growth promoting potential C. usual problem solving methods and coping mechanisms produce a solution D. anxiety or depression continue to increase 13. The nurse employs this approach in crisis intervention: A. problem-solving B. behavior modification C. role-playing D. nurse-patient relationship 14. Assessment data of the nurse include all the following EXCEPT: A. coping mechanisms B. situational support C. perception of the event D. repressed problems 15. The duration of crisis usually lasts several days and usually: A. 2 4 weeks B. 1 2 weeks C. 1 2 months

D. 4 6 weeks Situation - The community health nurse encounters special children in the community. 16. An individual with antisocial personality disorder lacks remorse, shame and guilt in going against the norms of society. Psychodynamically, this defect in the personality reflects a disturbance of the: A. ego B. super ego C. ego ideal D. id 17. The nurse teaches parents about childrens beginning concepts of right and wrong by emphasizing child rearing attitude and practices during the: A. school age B. toddler age C. infancy period D. latency period 18. It is BEST for parents to teach healthy interpersonal relationships to their children by: A. modeling to their children B. encouraging their children to attend secondary school C. encouraging their children at home to behave properly D. teaching their children good manners and right conduct 19. An important principle for the nurse to follow in interacting with retarded children is: A. seen that if the child appears contented, his needs are being met B. provide an environment appropriate to their development task as scheduled C. treat the child according to his chronological age D. treat the child according to his developmental level

20. Mental retardation is: A. a delay in normal growth and development caused by an inadequate environment B. a lack of development of sensory abilities C. a condition of subaverage intellectual functioning that originates during the developmental period and is associated with impairment in adaptive behavior D. a severe lag in neuromuscular development and motor abilities Situation - As a professional, it is imperative that the nurse is accountable to oneself hence the importance of personal and professional development. 21. Nurse: I feel personally involved with my clients problems demonstrates: A. counter transference B. empathy C. transference D. sympathy 22. The nurse has achieved self-awareness in which of the following verbalizations? A. every time people around me yell, I feel upset and withdrawn B. when the patient yelled at me I became speechless C. with the patients tone of voice and stare, I got reminded of how my father would be so angry and this made me anxious D. I thought it was rude for the patient to yell hence I kept quiet 23. An accepting attitude requires being: A. aware of ones biases B. tolerant of the faults of others C. non judgmental D. in control of tendency to blame

24. Self-awareness, knowledge and understanding of human behavior and communication skills define what is essential in caring for every nurse to be able to demonstrate: A. positive self-projection B. assertiveness C. therapeutic use of self D. self-mastery 25. Considering that man is by nature social, it is BEST for the nurse to gain self-awareness by: A. participating in intensive group experiences B. individual psychotherapy C. hypnotherapy D. writing an autobiography for self introspection Situation Ninety year old Purita is confined at the medical unit for respiratory ailment for which a breathing apparatus is prescribed for her to use while she sleeps. She refuses to wear continuously though she full understands the medical indication for it: 26. Which of these ethical principles can guide the nurse in her action? A. Beneficence B. Fidelity C. Autonomy D. Nonmaleficence 27. Purita has six children who already adults. They differ in their opinion whether or not to allow their mother to decide for her. The nurse would encourage family conference for: A. The eldest childs opinion to be given priority. B. Majority of the children to decide C. Allowing the medical staff to decide in their behalf D. Consensus building 28. Breathing treatments are to be given to Purita. In anticipation that Purita might refuse. Dinio, one of the children requests that he be the one to sign the consent in behalf of their mother. The nurse explains that Purita is rational in her thinking and which of this clients right must be regarding? A. Right to refuse treatment B. Right to privacy C. Right to informed consent D. Right of habeas consent 29. Which of these would be the nurses priority following the treatment principle of least restrictive alternative? A. One on one staffing B. Use of on site guard/watcher C. Physical restraint D. Seclusion 30. Purita talks about her joy in having responsible and accomplished children and recalls challenging career as a lawyer. She is demonstrating a sense of: A. Ego integrity B. Industry C. Generativity D. Autonomy Situation The supervising nurse received report that a staff nurse is displaying frequent irritation, anger, and even indifference toward clients and co-workers. 31. The initial action of the supervisor would be to: A. Post guidelines on proper decorum of nurse in the bulletin board. B. Write a memo of warning to the house C. Request anecdotal report from the nurses co-workers D. Call the nurse for a one on one conference 32. The nurse expressed increasing feelings of dissatisfaction. The supervising nurse intervenes therapeutically by taking the role of:

A. Administrator by relieving her of responsibilities B. Therapist by delving into the nurses internal conflict C. Counselor by actively listening D. Educator by reorienting her of role as a nurse. 33. Coupled with poor work performance, mental and physical fatigue and actual withdrawal from client contact and nursing duties. The nurse can be said to be suffering from: A. Psychotic anxiety B. Staff burn-out C. Personality maladjustment D. Neurotic depression 34. A priority in the nurses personal development would be to: A. Address her physical well-being B. Boost her self-confidence C. Provide social support D. Help her find value and meaning in her work 35. The most relevant professional program for her would be: A. Assertiveness training B. Stress management C. Group dynamics and team building D. Behavior modification

Situation A nurse assigned in the neurologic unit is taking of clients with varying degrees of generative disorders. 36. Mr. A with Myasthenia Gravis is having difficulty speaking. What communication strategies should the nurse avoid when interacting with Mr. A? A. Repeating what the client says for better understanding B. Using paper and pencil in communicating with the client C. Encouraging the client to speak slowly D. Encouraging the client to speak quickly 37. When planning for nursing care for Mr. B who has Parkinsons disease. Which of the following goals would be most appropriate? A. To improve muscle tone B. To start rehabilitation as much as possible C. TO treat the disease D. TO maintain optimal body function 38. For the past 10 years, Alma, 42 years old has had multiple sclerosis. Client with multiple sclerosis experience many different symptoms. As part of the rehabilitation planned for Alma, the nurse suggested therapy and hobbies to help her. A. Strengthen muscle coordination B. Establish routine C. Develop perseverance and motivation D. Establish good health habits 39. On his second day of hospitalization, Mr. Santos was unable to stand and is having difficulty swallowing and talking. Which of the following is the priority of the nurse in assisting Mr. Santos? A. To prevent bladder distention B. To prevent decubitus ulcer C. To prevent contracture D. To prevent aspiration pneumonia 40. The wife of a seventy two year old man with a diagnosis of Alzheimers disease begins to cry and tells the nurse, I could not understand my husband anymore. He has changed drastically. Which of the following responses of the nurse is MOST appropriate? A. The physician and the staff will make sure that your husband will be comfortable and safe here. B. This has been a difficult time for you. Let us walk and find a quiet place where we can talk. C. He will soon recover in his condition. D. You need not worry, we are doing the best we could.

Situation Annie has a morbid fear of heights. She asks the nurse what desensitization therapy is: 41. The accurate information of the nurse the goal of desensitization is: A. To help the clients relax and progressively work up a list of anxiety provoking situations through imagery B. To provide corrective emotional experiences through a one-to-one intensive relationship C. To help clients in a group therapy setting to take on specific roles and reenact in front of an audience, situations in which interpersonal conflict is involved. D. To help clients cope with their problems by learning behaviors that we are more functional and be better equipped to face reality and make decisions. 42. It is essential in desensitization for the patient to: A. Have a rapport with therapist B. Use deep breathing or another relaxation technique C. Assess ones self for the need of anxiolytic drug D. Work through unresolved unconsciousness conflicts 43. In this level of anxiety, cognitive capacity diminishes. Focus becomes limited and client experiences turned vision. Physical signs of anxiety become more pronounced. A. Severe anxiety B. Panic C. Mild anxiety D. Moderate anxiety 44. Anti-anxiety medication should be used with extreme caution because long term use can lead to. A. Parkinsonian like syndrome B. Hypertensive crisis C. Hepatic failure D. Risk of addiction 45. The nursing management of anxiety related with post traumatic stress disorder includes all of the following EXCEPT: A. Encourage participation in recreation or sport activities B. Reassurance clients safety while touching client C. Speak in calm soothing voice D. Remain with the client while fear level is high Situation The nurse is often met with the following situations when clients become angry and aggressive individual, the nurse should: 46. To maintain a therapeutic eye contact and body posture while interacting with angry and aggressive individual, the nurse should: A. keep an eye contact while staring at the client B. keep his/her hands behind his/her back or in ones pockets C. fold his/her arms across his/her chest D. keep an open posture, e.g. Hands by sides but palms turned outwards 47. During the pre-interaction phase of the N-P relationship, the nurse recognizes thisnormal INITIAL reaction to an assaultive or potentially assaultive person. A. To remain and cope with the incident B. Display empathy towards the patient C. To call for help from other members of the team D. To stay and fight or run away 48. Which of the following is an accurate way of reporting and recording an incident? A. When asked about his relationship with his father, client became anxious. B. When asked about his relationship with his father, client clenched his jaw/teeth, made a fist and turned away from the nurse. C. When asked about his relationship with his father, client was resistant to respond D. When asked about his relationship with his father, his anger was suppressed

49. To encourage thought, which of the following approaches is NOT therapeutic? A. Why do you feel angry? B. When do you usually feel angry? C. How do you usually express anger? D. What situations provoke you to be angry? 50. A patient grabs and about to throw it. The nurse best responds saying. A. Stop! Put that chair down. B. Dont be silly. C. Stop! The security will be here in a minute. D. Calm down.

NP5 Nursing Board Exam June 2008 Answer Key 'Nursing Care of Client with Physiological and Psychosocial Alteration'
100 Nursing Board Exam test questions of June 2008 Nurse Licensure Examination (NLE) Nursing Practice V Nursing Care of Client with Physiological and Psychosocial Alteration Medical and Surgical Nursing / Psychology PART 2 Board Exam test questions 1 - 50 June 2008
Situation A vehicle hit some pedestrian while waiting for a bus ride. Some of the victims suffered injuries in different part of the body. The victims were brought to the nearby hospital. One of the victims, Josephine was confirmed to have a fractured left arm. While waiting for plaster cast to be applied, Josephine appears to be anxious. 51. To reduce anxiety, the nurse teaches the procedure to the client, which of the following topics should NOT be included in the teaching plan? A. Leave the cast uncovered to promote drying B. Bear weight on the plaster for one hour. A stockinet will be placed over the left arm to be placed in the cast. C. Handle hardening cast with palm of hands. D. Trim and reshape finish cast with knife or cutter. 52. Cast was applied on Josephines left arm. In assessing the neurovascular status of the client, which of the following assessment findings should be reported to the physician? A. Pain on the left arm B. Swelling of the fingers C. Skin abrasion on the edges of the plaster cast D. Nail bed capillary refill time of 10 seconds 53. One of the victims a sixty year old woman sustained hip fracture. Prior to surgery, a Bucks extension traction is to be applied. The rationale of traction is primarily based the understanding that Bucks extension traction: A. Reduces muscle spasms and helps to immobilize the fracture B. Allows reduction of the fracture site for bone healing. C. Secures the fracture site to prevent damage to the muscle tissues D. Secures the fracture site for rigid immobilization 54. Philip was placed in skeletal leg traction with an overbed frame. He is not allowed to move from side to side. Which of the following nursing interventions is useful in maintaining. A. Assist the client by holding the trapeze and raising hips of the bed. B. Check the apparatus that weights hang free and knots in the rope are tied securely. C. Suspend a trapeze within easy reach of the client D. Support the affected extremity while the weights are removed. 55. To prevent complication when a child is in Bucks traction, the nurse should:

A. Clean the extremity and keep the skin dry B. Assess any skin and circulatory disturbances C. Clean the pin sites as necessary D. Provide high fiber small meals.

56. The following concept are true EXCEPT: A. Hostility is destructive B. Frustration develops in response to unmet needs, wants and desire C. Anger is incompatible with love D. Aggression can be expressed in a constructive as well as a destructive Manner 57. Carlo is acting out hostile and aggressive feeling by kicking the chairs in the room, the MOST effective way to deal with Carlos behavior is initially to: A. Set limits on the behavior by verbal command B. Administer PRN tranquilizer C. Remove the chairs from the room D. Restrain the patient and place him in the Isolation Room 58. Mrs. Dizon was visiting her son at the Psychiatry Ward. Which of the following items will the nurse NOT allow to brought inside the ward? A. String rosary bracelet B. Box of cake C. Bottle of coke D. Rubber shoes 59. Which of the following will probably be most therapeutic for a patient on a behavioral modification ward? A. If the client is agitated, discuss the feelings especially anger B. Insist to stop obscene language by verbal reprimand C. Give client support and positive feedback for controlling use of obscene language. D. Provide a punching bag as an alternative to express upset emotions. 60. Which of the following must be considered while planning activities for the depressed client? A. Activities which require exertion of energy B. Challenging activities to get him out of his depression C. Structured activities that the client can participate D. Variety of unstructured activities Situation - The nurse works with Mina to help her work through termination of the nurse-patient relationship. 61. Preparation for termination of the nurse-patient relationship begins during the: A. termination phase B. working phase C. pre-orientation phase D. orientation phase 62. Minas past reactions to ending relationships is withdrawal. The nurse assists her to practice better ways of coping termination by providing opportunities to: A. test new patterns of behavior B. plan for alternatives C. conceptualize her problem D. value and find meaning in experience 63. During the early part of the interaction, the nurse asked after a period of silence. Perhaps we would talk about my leaving. The nurse utilized which communication technique: A. encouraging B. suggesting C. focusing on client D. understanding 64. The longest and the most productive phase of the NPR is: A. Termination phase

B. Working phase C. Pre-orientation phase D. Orientation phase 65. The objection of the nurse-patient relationship is to provide an opportunity of the patient to: A. clarify problems B. develop insights C. have a corrective emotional experience D. develop interpersonal relationship Situation - As a program manager, the mental health psychiatric nurse is tasked to provide general patient management. 66. The nurse is aware that identifying the aspects of general patient management and identifying interventions for meeting these basic needs are distinctions of: A. psychotherapy B. therapeutic milieu C. pharmacotherapy D. behavior therapy 67. In order to get active participation of the clients to carry out the objective of the program, it is BEST for the nurse to conduct a/an: A. community meeting B. survey C. observation D. selective interview of patients 68. Through the nurses role modeling of effective communication, the clients learned new ways of dealing with authority figures. This gives the clients a venue to: A. communicate B. socialize C. identify their problems D. test new patterns of behavior 69. This element of communication facilitates evaluation of the program: A. receiver B. message C. sender D. feedback 70. The nurses style of leadership in milieu therapy is: A. autocratic B. laissez-faire C. democratic D. benevolent Situation - Loretta is a 28 years old, unemployed patient, admitted to the psychiatric unit with a diagnosis of chronic undifferentiated schizophrenia. She described herself as the Virgin Mary and her mission is to propagate peace. She was observed laughing and talking to herself. Her thought processes were profoundly disorganized. She was also fearful and suspicious of others. 71. Lorettas claim of being the Virgin Mary is a/an: A. imagination B. delusion C. hallucination D. obsession 72. Her mission to propagate peace is: A. a serious call for a need of transformation B. an attempt to overcome low self-esteem C. realistic and laudable D. an advocacy that she can participate in 73. Which of this intervention would NOT be therapeutic in decreasing Lorettas anxiety?

A. joke about her thought to help her feel at ease B. listen to her thoughts and feelings C. simply accept D. do not convince her that her perception is unreal 74. The nurse evaluates that Lorettas ready for a rehabilitation program when she: A. approaches the nurse at frequent intervals B. ceases to talk about Virgin Mary C. goes to the coffee shop alone more often D. carries a book The Purpose Driven Life 75. The nurse is leading a group meeting of patients to prepare them to be discharged. An appropriate goal for the group members is to develop: A. attitudes of society towards the mentally ill B. skills for maintaining daily living C. awareness of interpersonal patterns of interactions D. insight into personal problems Situation - A group of adult chronic schizophrenic patients were recommended to undergo social skills training. 76. The following are the objectives of a social skills training program EXCEPT: A. explore deep seated intrapsychic conflicts B. practice skills in relating with people C. help build self esteem and self confidence D. develop and practice general coping skills 77. Social skills training is NOT primarily indicated for psychiatric patients who are: A. in acute stage of illness B. having difficulties starting and maintaining interpersonal relationships C. having chronic episodes of stress and anxiety while interacting with others D. experiencing recurrence of symptoms in front of particular people or among people in general 78. The focus of the group interaction is here and now. An appropriate topic would be: A. ways to celebrate Valentines Day in February B. how to spend the summer vacation C. an unforgettable experience as a child D. how to tell a joke 79. An appropriate technique for the participants to practice how to communicate effectively is through/a: A. lecture B. seminar C. role play D. psychodrama 80. Considering that it is BEST to learn by example, it is MOST practical to: A. model good social skills throughout the session B. relate successful past experiences C. invite a resource person D. watch a movie Situation - Andy, 30 years old, was admitted to the Psychiatric Ward because of religious preoccupation, deterioration in self-care and disturbed thoughts. He believes that he has committed a lot of sins. He is threatened by people reaching out to him. His fasting for several days was not sufficient for him to feel forgiven. 81. Andy is demonstrating: A. religious delusion B. delusion of grandeur C. somatic delusion D. delusion of being controlled 82. A delusion is: A. psychomotor disturbance B. mood disturbance

C. disturbance of thought D. disturbance of perception 83. The nursing goal for Andy is to: A. have him see a priest for confession B. encourage him to pray to atone for his sins C. help him develop a positive self image D. socialize him with a group to keep him in touch with reality 84. As Andy talks about his sins that he believes make people look down upon him. It is BEST to: A. agree with him and sympathize how sinful he has really been B. explore the nature of his sins C. explain that he is depreciating himself too much D. acknowledge how he feels and focus on reality oriented topics 85. Which of the following drugs will most likely to decrease Andys hallucination? A. Chlorpromazine (Thorazine) B. Chlomipramine (Anafranil) C. Fluoxetine (Prozac) D. Carbamazepine (Tegretol) 86. Which of this client situation appropriately illustrate horticulture therapy? A. the therapist brings bongos, tambourine, and bells and encourages client participation B. Tommy, Karen, Jon and Pia play scrabble every night after supper C. every afternoon, Vic goes in the garden where he work with plants, seedlings, tree planting and watering them D. Paul finds sketching relaxing and rewarding 87. The beginning professional nurse can do mental health counseling with the following clients EXCEPT: A. Actively psychotic patients B. Out of school adolescents C. Parents with child rearing concerns D. School children with behavioral problem 88. A professional responsibility of the mental health psychiatric nurse is to provide a safe and therapeutic environment. This is BEST reflected in: A. restraining patients who violates policies and do not follow schedule of activities. B. maintaining a closed door policy to prevent patients from absconding. C. keeping a restrictive environment to prevent patients from becoming assaultive and hostile D. ensuring physical safety and maintaining therapeutic attitude towards the patients 89. The foundation of the therapeutic process is the therapeutic relationship. What is the essential component that the nurse must bring to the relationship? A. humor B. empathy C. reframing D. confrontation 90. Which of these people, the highest in population groups that would need priority mental health therapy? A. adults going through active skills B. single elderly with no social support C. young professionals entering the workplace D. women preparing for overseas employment Situation - The nurse observed that Marie, age 28 years old had not been participating in activity therapies. 91. Which of the following remarks from the nursing attendants indicates a need for further teaching and observation? A. Marie made no response to an invitation to play B. Marie preferred to sit at the bench and watch the ballgame

C. Marie is aloof and indifferent to co-patients D. Marie read a book while other patients played a ballgame 92. Marie said I dont like to be a part of it. Look, they are being laughed at because they are singing and acting like children. The nurse notes that in order for the activity therapy to be therapeutic: A. the staff should decide solely what activities to be done and what rules apply B. patients should be allowed solely to decide what they want to do on their own C. ballgames should be limited to male patients only D. age and needs of patient should be considered in the choice of games 93. To encourage active participation among patients, it is BEST to plan activities they can engage in through a: A. one to one interaction B. community meeting C. checklist D. feedback evaluation forum 94. In planning activities for the patients, it is essential to consider FOREMOST: A. safety and security B. variety and fun C. novelty and creativity D. excitement and challenge 95. Adults, singing and acting like children is a form of: A. displacement B. regression C. sublimation D. compensation Situation Nursing care for the elderly 96. In planning care for a patient with Parkinsons disease, which of these nursing diagnoses should have priority? A. potential for injury B. altered nutritional state C. ineffective coping D. altered mood state 97. The frequent use of the older clients name by the nurse is MOST effective in alleviating which of the following responses to old age? A. Loneliness B. Suspicion C. Grief D. Confusion 98. An elderly who has lots of regrets, unhappy and miserable is experiencing: A. Crisis B. Despair C. Loss D. Ambivalence 99. The pre-morbid personality of a schizophrenic elderly client is: A. Schizoid B. Extrovert C. Ambivert D. Cycloid 100. Schizophrenia is a/an: A. Anxiety disorder B. Neurosis C. Psychosis D. Personality disoder

You might also like